Hough and Nielsen [HoNi19] proved that at least one modulus must be divisible by either $2$ or $3$. A simpler proof of this fact was provided by Balister, Bollobás, Morris, Sahasrabudhe, and Tiba [BBMST22].
Selfridge has shown (as reported in [Sc67]) that such a covering system exists if a covering system exists with moduli $n_1,\ldots,n_k$ such that no $n_i$ divides any other $n_j$ (but the latter has been shown not to exist, see [586]).
The sequence of such numbers is A006286 in the OEIS.
Granville and Soundararajan [GrSo98] have conjectured that at most $3$ powers of 2 suffice for all odd integers, and hence at most $4$ powers of $2$ suffice for all even integers. (The restriction to odd integers is important here - for example, Bogdan Grechuk has observed that $1117175146$ is not the sum of a prime and at most $3$ powers of $2$, and pointed out that parity considerations, coupled with the fact that there are many integers not the sum of a prime and $2$ powers of $2$ (see [9]) suggest that there exist infinitely many even integers which are not the sum of a prime and at most $3$ powers of $2$).
Granville and Soundararajan [GrSo98] have proved that this is very related to the problem of finding primes $p$ for which $2^p\equiv 2\pmod{p^2}$ (for example this conjecture implies there are infinitely many such $p$).
Erdős often asked this under the weaker assumption that $n$ is not divisible by $4$. Erdős thought that proving this with two powers of 2 is perhaps easy, and could prove that it is true (with a single power of two) for almost all $n$.
Is it true that \[\sum_{n\in A}\frac{1}{n}<\infty?\]
An example of an $A$ with this property where \[\liminf \frac{\lvert A\cap\{1,\ldots,N\}\rvert}{N^{1/2}}\log N>0\] is given by the set of $p^2$, where $p\equiv 3\pmod{4}$ is prime.
Elsholtz and Planitzer [ElPl17] have constructed such an $A$ with \[\lvert A\cap\{1,\ldots,N\}\rvert\gg \frac{N^{1/2}}{(\log N)^{1/2}(\log\log N)^2(\log\log\log N)^2}.\]
Schoen [Sc01] proved that if all elements in $A$ are pairwise coprime then \[\lvert A\cap\{1,\ldots,N\}\rvert \ll N^{2/3}\] for infinitely many $N$. Baier [Ba04] has improved this to $\ll N^{2/3}/\log N$.
For the finite version see [13].
Is it true that for all $\epsilon>0$ and large $N$ \[\lvert \{1,\ldots,N\}\backslash B\rvert \gg_\epsilon N^{1/2-\epsilon}.\] Is it true that \[\lvert \{1,\ldots,N\}\backslash B\rvert =o(N^{1/2})?\]
Erdös and Freud investigated the finite analogue in 'a recent Hungarian paper', proving that there exists $A\subseteq \{1,\ldots,N\}$ such that the number of integers not representable in exactly one way as the sum of two elements from $A$ is $<2^{3/2}N^{1/2}$, and suggest the constant $2^{3/2}$ is perhaps best possible.
Tao [Ta23] has proved that this series does converge assuming a strong form of the Hardy-Littlewood prime tuples conjecture.
In [Er98] Erdős further conjectures that \[\sum_{n=1}^\infty (-1)^n \frac{1}{n(p_{n+1}-p_n)}\] converges and \[\sum_{n=1}^\infty (-1)^n \frac{1}{p_{n+1}-p_n}\] diverges. He further conjectures that \[\sum_{n=1}^\infty (-1)^n \frac{1}{n(p_{n+1}-p_n)(\log\log n)^c}\] converges for every $c>0$, and reports that he and Nathanson can prove this for $c>2$ (and conditionally for $c=2$).
Blecksmith, Erdős, and Selfridge [BES99] proved that the number of such primes is \[\ll_A \frac{x}{(\log x)^A}\] for every $A>0$, and Elsholtz [El03] improved this to \[\ll x\exp(-c(\log\log x)^2)\] for every $c<1/8$.
Are there infinitely many practical $m$ such that \[h(m) < (\log\log m)^{O(1)}?\] Is it true that $h(n!)<n^{o(1)}$? Or perhaps even $h(n!)<(\log n)^{O(1)}$?
The sequence of practical numbers is A005153 in the OEIS.
Conjectured by Bollobás and Erdős [BoEr76], who proved the existence of such a graph with $(1/8+o(1))n^2$ many edges. Solved by Fox, Loh, and Zhao [FLZ15], who proved that for every $n\geq 1$ there exists a graph on $n$ vertices with $\geq n^2/8$ many edges, containing no $K_4$, whose largest independent set has size at most \[ \ll \frac{(\log\log n)^{3/2}}{(\log n)^{1/2}}n.\]
See also [615].
In [Er92b] Erdős asks, more generally, if a graph on $(2k+1)n$ vertices in which every odd cycle has size $\geq 2k+1$ can be made bipartite by deleting at most $n^2$ edges.
In [Er92b] and [Er97f] Erdős asks more generally: if $r\geq 5$ is odd and a graph has $rn$ vertices and the smallest odd cycle has size $r$ then is the number of cycles of size $r$ at most $n^{r}$?
Tenenbaum asked the weaker variant (still open) where for every $\epsilon>0$ there is some $k=k(\epsilon)$ such that at least $1-\epsilon$ density of all integers have a divisor of the form $a+k$ for some $a\in A$.
Can the bound $O(\log N)$ be achieved? Must such an $A$ satisfy \[\liminf \frac{\lvert A\cap\{1,\ldots,N\}\rvert}{\log N}> 1?\]
Erdős [Er54] proved that such a set $A$ exists with $\lvert A\cap\{1,\ldots,N\}\rvert\ll (\log N)^2$ (improving a previous result of Lorentz [Lo54] who achieved $\ll (\log N)^3$). Wolke [Wo96] has shown that such a bound is almost true, in that we can achieve $\ll (\log N)^{1+o(1)}$ if we only ask for almost all integers to be representable.
The answer to the third question is yes: Ruzsa [Ru98c] has shown that we must have \[\liminf \frac{\lvert A\cap\{1,\ldots,N\}\rvert}{\log N}\geq e^\gamma\approx 1.781.\]
This is extremely false, as shown by Konieczny [Ko15], who both constructs an explicit permutation with $S(\pi) \geq n^2/4$, and also shows that for a random permutation we have \[S(\pi)\sim \frac{1+e^{-2}}{4}n^2.\]
Ruzsa has observed that this follows immediately from the stronger fact proved by Plünnecke [Pl70] that (under the same assumptions) \[d_S(A+B)\geq \alpha^{1-1/k}.\]
For all sufficiently large $N$, if $A\sqcup B=\{1,\ldots,2N\}$ is a partition into two equal parts, so that $\lvert A\rvert=\lvert B\rvert=N$, then there is some $x$ such that the number of solutions to $a-b=x$ with $a\in A$ and $b\in B$ is at least $cN$.
Can a lacunary set $A\subset\mathbb{N}$ be an essential component?
The Schnirelmann density is defined by \[d_s(A) = \inf_{N\geq 1}\frac{\lvert A\cap\{1,\ldots,N\}\rvert}{N}.\]
This is a stronger propery than $B$ being an essential component (see [37]). Linnik [Li42] gave the first construction of an essential component which is not an additive basis.
Indeed, we must trivially have $\sum_{d|n_k}1/d \geq k$, or else there is a greedy colouring as a counterexample. Since $\prod_{p}(1+1/p^2)$ is finite we must have $\prod_{p|n_k}(1+1/p)\gg k$. To achieve the minimal $\prod_{p|n_k}p$ we take the product of primes up to $T$ where $\prod_{p\leq T}(1+1/p)\gg k$; by Mertens theorems this implies $T\geq C^{k}$ for some constant $C>1$, and hence $n_k\geq \prod_{p\mid n_k}p\geq \exp(cC^k)$ for some $c>0$.
Solved by Tao [Ta23b], who proved that \[ \lvert A\rvert \leq \left(1+O\left(\frac{(\log\log x)^5}{\log x}\right)\right)\pi(x).\]
In [Er95c] Erdős further asks about the situation when $\phi(a_1)\leq \cdots \leq \phi(a_t)$.
See also [694].
Erdős and Szemerédi proved that there exist arbitrarily large sets $A$ such that the integers which are the sum or product of distinct elements of $A$ is at most \[\exp\left(c (\log \lvert A\rvert)^2\log\log\lvert A\rvert\right)\] for some constant $c>0$.
See also [52].
Erdős later asked ([Er92b] and [Er95]) if the conjecture remains true provided $N\geq (1+o(1))p_k^2$ (or, in a weaker form, whether it is true for $N$ sufficiently large depending on $k$).
See also [534].
The lower density of the set can be $0$ since there are graphs of arbitrarily large chromatic number and girth.
See also [65].
A stronger form was established by Gao, Huo, and Ma [GaHuMa21], who proved that if a graph $G$ has chromatic number $\chi(G)\geq 2k+3$ then $G$ contains cycles of $k+1$ consecutive odd lengths.
He, Ma, and Yang [HeMaYa21] have proved this conjecture when $n=q^2+q+1$ for some even integer $q$.
Every graph with chromatic number $\aleph_1$ contains all sufficiently large odd cycles (which have chromatic number $3$), see [594]. This was proved by Erdős, Hajnal, and Shelah [EHS74]. Erdős wrote [Er87] that 'probably' every graph with chromatic number $\aleph_1$ contains as subgraphs all graphs with chromatic number $4$ with sufficiently large girth.
David Penman has observed that this is certainly true if the graph has uncountable chromatic number, since by a result of Erdős and Hajnal [ErHa66] such a graph must contain arbitrarily large finite complete bipartite graphs (see also Theorem 3.17 of Reiher [Re24]).
Zach Hunter has observed that this follows from the work of Liu and Montgomery [LiMo20]: if $G$ has infinite chromatic number then, for infinitely many $r$, it must contain some finite connected subgraph $G_r$ with chromatic number $r$ (via the de Bruijn-Erdős theorem [dBEr51]). Each $G_r$ contains some subgraph $H_r$ with minimum degree at least $r-1$, and hence via Theorem 1.1 of [LiMo20] there exists some $\ell_r\geq r^{1-o(1)}$ such that $H_r$ contains a cycle of every even length in $[(\log \ell)^8,\ell]$.
See also [64].
See also the entry in the graphs problem collection.
See also [57].
Pratt [Pr24] has proved this is irrational, conditional on a uniform version of the prime $k$-tuples conjecture.
Tao has observed that this is a special case of [257], since \[\sum_{n\geq 2}\frac{\omega(n)}{2^n}=\sum_p \frac{1}{2^p-1}.\]
See also [922] and the entry in the graphs problem collection.
The answer is yes, proved by Gruslys and Letzter [GrLe20].
In [Er97d] Erdős also asks for a lower bound for the count of edge-disjoint monochromatic triangles in single colour (the colour chosen to maximise this quantity), and speculates that the answer is $\geq cn^2$ for some constant $c>1/24$.
Are there infinitely many graphs $G$ which are not Ramsey size linear but such that all of its subgraphs are?
Estimate $f_c(n)$. In particular, is it true that $f_c(n)>n^{\epsilon}$ for some $\epsilon>0$? Or $f_c(n)\gg \log n$?
Edwards (unpublished) and Khadziivanov and Nikiforov [KhNi79] proved independently that $f_c(n) \geq n/6$ when $c>1/4$ (see [905]).
Fox and Loh [FoLo12] proved that \[f_c(n) \leq n^{O(1/\log\log n)}\] for all $c<1/4$, disproving the first conjecture of Erdős.
The best known lower bounds for $f_c(n)$ are those from Szemerédi's regularity lemma, and as such remain very poor.
See also [600] and the entry in the graphs problem collection.
A split graph is one where the vertices can be split into a clique and an independent set. Every split graph is chordal. Chen, Erdős, and Ordman [CEO94] have shown that any split graph can be partitioned into $\frac{3}{16}n^2+O(n)$ many cliques.
Prove that $f(n)=o(2^n)$.
Prove that $f(n)/2^{n/2}\to \infty$.
One can also ask about the existence and value of $\lim f(n)^{1/n}$.
Even stronger, is there some $c>0$ such that, for all large $k$, $R(G)>cR(k)$ for every graph $G$ with chromatic number $\chi(G)=k$?
Since $R(k)\leq 4^k$ this is trivial for $\epsilon\geq 3/4$. Yuval Wigderson points out that $R(G)\gg 2^{k/2}$ for any $G$ with chromatic number $k$ (via a random colouring), which asymptotically matches the best-known lower bounds for $R(k)$.
This problem is #12 and #13 in Ramsey Theory in the graphs problem collection.
Szemerédi conjectured (see [Er97e]) that this stronger variant remains true if we only assume that no three points are on a line, and proved this with the weaker bound of $n/3$.
See also [660].
Edelsbrunner and Hajnal [EdHa91] have constructed $n$ such points with $2n-7$ pairs distance $1$ apart. (This disproved an early stronger conjecture of Erdős and Moser, that the true answer was $\frac{5}{3}n+O(1)$.)
It is easy to see that $h_c(n) \ll_c n^{1/2}$, and Erdős originally suggested that perhaps a similar lower bound $h_c(n)\gg_c n^{1/2}$ holds. Zach Hunter has pointed out that this is false, even replacing $>3$ points on each line with $>k$ points: consider the set of points in $\{1,\ldots,m\}^d$ where $n\approx m^d$. These intersect any line in $\ll_d n^{1/d}$ points, and have $\gg_d n^2$ many pairs of points each of which determine a line with at least $k$ points. This is a construction in $\mathbb{R}^d$, but a random projection into $\mathbb{R}^2$ preserves the relevant properties.
This construction shows that $h_c(n) \ll n^{1/\log(1/c)}$.
See also [99].
In [Er75h] Erdős also asks how many such unit circles there must be if the points are in general position.
It is trivial from the Cauchy-Schwarz inequality that $f(k^2)=k$. Erdős also asks for which $n$ is it true that $f(n+1)=f(n)$.
It is easy to see that $f(k^2+1)\geq k$, by first dividing the unit square into $k^2$ smaller squares of side-length $1/k$, and then replacing one square by two smaller squares of side-length $1/2k$. Halász [Ha84] gives a construction that shows $f(k^2+2)\geq k+\frac{1}{k+1}$, and in general, for any $c\geq 1$, \[f(k^2+2c+1)\geq k+\frac{c}{k}\] and \[f(k^2+2c)\geq k+\frac{c}{k+1}.\] Halász also considers the variants where we replace a square by a parallelogram or triangle.
Erdős and Soifer [ErSo95] and Campbell and Staton [CaSt05] have conjectured that, in general, for any integer $-k<c<k$, $f(k^2+2c+1)=k+\frac{c}{k}$, and proved the corresponding lower bound. Praton [Pr08] has proved that this general conjecture is equivalent to $f(k^2+1)=k$.
Baek, Koizumi, and Ueoro [BKU24] have proved $g(k^2+1)=k$, where $g(\cdot)$ is defined identically to $f(\cdot)$ with the additional assumption that all squares have sides parallel to the sides of the unit square. More generally, they prove that $g(k^2+2c+1)=k+c/k$ for any $-k<c<k$, which determines all values of $g(\cdot)$.
In [Er79b] Erdős also asks whether \[\lim_{k\to \infty}\frac{f(k,r+1)}{f(k,r)}=\infty.\]
See also the entry in the graphs problem collection and [740] for the infinitary version.
A theorem of de Bruijn and Erdős [dBEr51] implies that, if $G$ has infinite chromatic number, then $G$ has a finite subgraph of chromatic number $n$ for every $n\geq 1$.
In [Er95d] Erdős suggests this is true, although such an $F$ must grow faster than the $k$-fold iterated exponential function for any $k$.
What is the behaviour of $h_G(n)$? Is it true that $h_G(n)/n\to \infty$ for every graph $G$ with chromatic number $\aleph_1$?
On the other hand, Erdős, Hajnal, and Szemerédi proved that there is a $G$ with chromatic number $\aleph_1$ such that $h_G(n)\ll n^{3/2}$. In [Er81] Erdős conjectured that this can be improved to $\ll n^{1+\epsilon}$ for every $\epsilon>0$.
See also [74].
See also the entry in the graphs problem collection - on this site the problem replaces transitive tournament with directed path, but Zach Hunter and Raphael Steiner have a simple argument that proves, for this alternative definition, that $k(n,m)=(n-1)(m-1)$.
The Chebyshev polynomials show that $n^2/2$ is best possible here. Erdős originally conjectured this without the $o(1)$ term but Szabados observed that was too strong. Pommerenke [Po59a] proved an upper bound of $\frac{e}{2}n^2$.
Eremenko and Lempert [ErLe94] have shown this is true, and in fact Chebyshev polynomials are the extreme examples.
Wagner [Wa88] proves, for $n\geq 3$, the existence of such polynomials with \[\mu(A) \ll_\epsilon (\log\log n)^{-1/2+\epsilon}\] for all $\epsilon>0$.
Estimate $h(n)$ as well as possible.
The answer is no, as independently shown by Schipperus [Sc99] (published in [Sc10]) and Darby [Da99].
For example, Larson [La00] has shown that this is false when $\alpha=\omega^{\omega^2}$ and $n=5$. There is more background and proof sketches in Chapter 2.9 of [HST10], by Hajnal and Larson.
Erdős (and independently Hall [Ha96] and Montgomery) also asked about $F(N)$, the size of the largest $A\subseteq\{1,\ldots,N\}$ such that the product of no odd number of $a\in A$ is a square. Ruzsa [Ru77] observed that $1/2<\lim F(N)/N <1$. Granville and Soundararajan [GrSo01] proved an asymptotic \[F(N)=(1-c+o(1))N\] where $c=0.1715\ldots$ is an explicit constant.
This problem was answered in the negative by Tao [Ta24], who proved that for any $k\geq 4$ there is some constant $c_k>0$ such that $F_k(N) \leq (1-c_k+o(1))N$.
See also [888].
See also [125].
Does $A+B$ have positive density?
If $C=A+B$ then Melfi [Me01] showed $\lvert C\cap[1,x]\rvert \gg x^{0.965}$ and Hasler and Melfi [HaMe24] improved this to $\lvert C\cap [1,x]\rvert \gg x^{0.9777}$. Hasler and Melfi also show that the lower density of $C$ is at least \[\frac{1015}{1458}\approx 0.69616.\]
See also [124].
The obvious probabilistic construction (randomly colour the edges red/blue independently uniformly at random) yields a 2-colouring of the edges of $K_N$ such every set on $n$ vertices contains a red triangle and a blue triangle (using that every set of $n$ vertices contains $\gg n^2$ edge-disjoint triangles), provided $N \leq C^n$ for some absolute constant $C>1$. This implies $R(n;3,2) \geq C^{n}$, contradicting the conjecture.
Perhaps Erdős had a different problem in mind, but it is not clear what that might be. It would presumably be one where the natural probabilistic argument would deliver a bound like $C^{\sqrt{n}}$ as Erdős and Gyárfás claim to have achieved via the probabilistic method.
How large can the chromatic number and clique number of this graph be? In particular, can the chromatic number be infinite?
See also [213].
What is the order of growth of $f(n)$? Does $f(n)/\sqrt{n}\to \infty$?
Simonovits observed that the subsets of $[3m-1]$ of size $m$, two sets joined by edge if and only if they are disjoint, forms a triangle-free graph of diameter $2$ which is regular of degree $\binom{2m-1}{m}$. This construction proves that \[f(n) \leq n^{(1+o(1))\frac{2}{3H(1/3)}}=n^{0.7182\cdots},\] where $H(x)$ is the binary entropy function. In [Er97b] Erdős encouraged the reader to try and find a better construction.
In this note Alon provides a simple construction that proves $f(n) \ll \sqrt{n\log n}$: take a triangle-free graph with independence number $\ll \sqrt{n\log n}$ (the existence of which is the lower bound in [165]) and add edges until it has diameter $2$; the neighbourhood of any set is an independent set and hence the maximum degree is still $\ll \sqrt{n\log n}$.
Hanson and Seyffarth [HaSe84] proved that $f(n)\leq (\sqrt{2}+o(1))\sqrt{n}$ using a Cayley graph on $\mathbb{Z}/n\mathbb{Z}$, with the generating set given by some symmetric complete sum-free set of size $\sim \sqrt{n}$. An alternative construction of such a complete sum-free set was given by Haviv and Levy [HaLe18].
Füredi and Seress [FuSe94] proved that $f(n)\leq (\frac{2}{\sqrt{3}}+o(1))\sqrt{n}$.
The precise asymptotics of $f(n)$ are unknown; Alon believes that the truth is $f(n)\sim \sqrt{n}$.
Can $G$ be made into a triangle-free graph with diameter $2$ by adding at most $\delta n^2$ edges?
In this note Alon solves this problem in a strong form, in particular proving that a triangle-free graph on $n$ vertices with maximum degree $<n^{1/2-\epsilon}$ can be made into a triangle-free graph with diameter $2$ by adding at most $O(n^{2-\epsilon})$ edges.
See also [618].
In [Er82c] he further conjectures that, if $m,k$ are fixed and $n$ is sufficiently large, then there must be at least $k$ distinct primes $p$ such that \[p\mid m(m+1)\cdots (m+n)\] and yet $p^2$ does not divide the right-hand side.
See also [364].
The existence of such progressions for small $k$ has been verified for $k\leq 10$, see the Wikipedia page. It is open, even for $k=3$, whether there are infinitely many such progressions.
See also [219].
The weaker conjecture that there exists some $c>0$ such that $(2-c)\Delta^2$ sets suffice was proved by Molloy and Reed [MoRe97], who proved that $1.998\Delta^2$ sets suffice (for $\Delta$ sufficiently large). This was improved to $1.93\Delta^2$ by Bruhn and Joos [BrJo18] and to $1.835\Delta^2$ by Bonamy, Perrett, and Postle [BPP22]. The best bound currently available is \[1.772\Delta^2,\] proved by Hurley, de Joannis de Verclos, and Kang [HJK22].
Erdős and Nešetřil also asked the easier problem of whether $G$ containing at least $\tfrac{5}{4}\Delta^2$ many edges implies $G$ containing two strongly independent edges. This was proved independently by Chung-Trotter and Gyárfás-Tuza.
Does $c(n)^{1/n}\to \alpha$ for some $\alpha <2$?
Solved by Bradač [Br24], who proved that $\alpha=\lim c(n)^{1/n}$ exists and \[\alpha \leq 2^{H(1/3)}=1.8899\cdots,\] where $H(\cdot)$ is the binary entropy function. Seymour's construction proves that $\alpha\geq 3^{1/3}=1.442\cdots$. Bradač conjectures that this lower bound is the true value of $\alpha$.
This problem is #17 in Ramsey Theory in the graphs problem collection.
Prove that for every fixed $0\leq \alpha \leq 1/2$, as $n\to\infty$, \[F(n,\alpha)\sim c_\alpha \log n\] for some constant $c_\alpha$.
This problem is #9 in Ramsey Theory in the graphs problem collection. See also [800].
Is \[\lim_{k\to \infty}\frac{f(k)}{\log W(k)}=\infty\] where $W(k)$ is the van der Waerden number?
Moreira [Mo17] has proved that in any finite colouring of $\mathbb{N}$ there exist $x,y$ such that $\{x,x+y,xy\}$ are all the same colour.
Alweiss [Al23] has proved that, in any finite colouring of $\mathbb{Q}\backslash \{0\}$ there exist arbitrarily large finite $A$ such that all sums and products of distinct elements in $A$ are the same colour. Bowen and Sabok [BoSa22] had proved this earlier for the first non-trivial case of $\lvert A\rvert=2$.
Sets known to be Ramsey include vertices of $k$-dimensional rectangles [EGMRSS73], non-degenerate simplices [FrRo90], trapezoids [Kr92], and regular polygons/polyhedra [Kr91].
Proved by Sárkzözy [Sa85] for all sufficiently large $n$, and by Granville and Ramaré [GrRa96] for all $n\geq 5$.
More generally, if $f(n)$ is the largest integer such that, for some prime $p$, we have $p^{f(n)}$ dividing $\binom{2n}{n}$, then $f(n)$ should tend to infinity with $n$. Can one even disprove that $f(n)\gg \log n$?
Is it true that, for every $\mathcal{F}$, there exists $G\in\mathcal{F}$ such that \[\mathrm{ex}(n;G)\ll_{\mathcal{F}}\mathrm{ex}(n;\mathcal{F})?\]
This is trivially true if $\mathcal{F}$ does not contain any bipartite graphs, since by the Erdős-Stone theorem if $H\in\mathcal{F}$ has minimal chromatic number $r\geq 2$ then \[\mathrm{ex}(n;H)=\mathrm{ex}(n;\mathcal{F})=\left(\frac{r-2}{r-1}+o(1)\right)\binom{n}{2}.\] Erdős and Simonovits observe that this is false for infinite families $\mathcal{F}$, e.g. the family of all cycles.
See also [575] and the entry in the graphs problem collection.
A construction due to Pyber, Rödl, and Szemerédi [PRS95] shows that this is best possible.
The best bound available is due to Bucić and Montgomery [BM22], who prove that $O(n\log^*n)$ many cycles and edges suffice, where $\log^*$ is the iterated logarithm function.
Conlon, Fox, and Sudakov [CFS14] proved that $O_\epsilon(n)$ cycles and edges suffice if $G$ has minimum degree at least $\epsilon n$, for any $\epsilon>0$.
See also [583].
The answer is yes, which is a corollary of the density Hales-Jewett theorem, proved by Furstenberg and Katznelson [FuKa91].
See also [789].
Erdős and Graham asked this with just any $k$-term arithmetic progression in blue (not necessarily with distance $1$), but Alon has pointed out that in fact no such $k$ exists: in any red/blue colouring of the integer points on a line either there are two red points distance $1$ apart, or else the set of blue points and the same set shifted by $1$ cover all integers, and hence by van der Waerden's theorem there are arbitrarily long blue arithmetic progressions.
It seems most likely, from context, that Erdős and Graham intended to restrict the blue arithmetic progression to have distance $1$ (although they do not write this restriction in their papers).
This is false; Kovač [Ko23] provides an explicit (and elegantly simple) colouring using 25 colours such that no colour class contains the vertices of a rectangle of area $1$. The question for parallelograms remains open.
In the same article Rödl also proved a lower bound for this problem, constructing, for all $n$, a $2$-colouring of $\binom{\{2,\ldots,n\}}{2}$ such that if $X\subseteq \{2,\ldots,n\}$ is such that $\binom{X}{2}$ is monochromatic then \[\sum_{x\in X}\frac{1}{\log x}\ll \log\log\log n.\]
This bound is best possible, as proved by Conlon, Fox, and Sudakov [CFS13], who proved that, if $n$ is sufficiently large, then in any $2$-colouring of $\binom{\{2,\ldots,n\}}{2}$ there exists some $X\subset \{2,\ldots,n\}$ such that $\binom{X}{2}$ is monochromatic and \[\sum_{x\in X}\frac{1}{\log x}\geq 2^{-8}\log\log\log n.\]
This problem is equivalent to one on 'abelian squares' (see [231]). In particular $A$ can be interpreted as an infinite string over an alphabet with $d$ letters (each letter describining which of the $d$ possible steps is taken at each point). An abelian square in a string $s$ is a pair of consecutive blocks $x$ and $y$ appearing in $s$ such that $y$ is a permutation of $x$. The connection comes from the observation that $p,q,r\in A\subset \mathbb{R}^d$ form a three-term arithmetic progression if and only if the string corresponding to the steps from $p$ to $q$ is a permutation of the string corresponding to the steps from $q$ to $r$.
This problem is therefore equivalent to asking for which $d$ there exists an infinite string over $\{1,\ldots,d\}$ with no abelian squares. It is easy to check that in fact any finite string of length $7$ over $\{1,2,3\}$ contains an abelian square.
An infinite string without abelian squares was constructed when $d=4$ by Keränen [Ke92]. We refer to a recent survey by Fici and Puzynina [FiPu23] for more background and related results, and a blog post by Renan for an entertaining and educational discussion.
These bounds were improved by Croot [Cr03b] who proved \[\frac{N}{L(N)^{\sqrt{2}+o(1)}}< f(N)<\frac{N}{L(N)^{1/6-o(1)}},\] where $f(N)=\exp(\sqrt{\log N\log\log N})$. These bounds were further improved by Chen [Ch05] and then by de la Bretéche, Ford, and Vandehey [BFV13] to \[\frac{N}{L(N)^{1+o(1)}}<f(N) < \frac{N}{L(N)^{\sqrt{3}/2+o(1)}}.\] The latter authors conjecture that the lower bound here is the truth.
That is, for all $d\mid n$ with $d>1$ there is an associated $a_d$ such that every integer is congruent to some $a_d\pmod{d}$, and if there is some integer $x$ with \[x\equiv a_d\pmod{d}\textrm{ and }x\equiv a_{d'}\pmod{d'}\] then $(d,d')=1$.
Adenwalla [Ad25] has proved there are no such $n$.
In general, for any $n$ one can try to choose such $a_d$ to maximise the density of integers so covered, and ask what this density is. This was also investigated by Adenwalla [Ad25].
See also [851].
Is it true that, for almost all $x$, for sufficiently large $n$, we have \[R_{n+1}(x)=R_n(x)+\frac{1}{m},\] where $m$ is minimal such that $m$ does not appear in $R_n(x)$ and the right-hand side is $<x$? (That is, are the best underapproximations eventually always constructed in a 'greedy' fashion?)
Without the 'eventually' condition this can fail for some rational $x$ (although Erdős [Er50b] showed it holds without the eventually for rationals of the form $1/m$). For example \[R_1(\tfrac{11}{24})=\frac{1}{3}\] but \[R_2(\tfrac{11}{24})=\frac{1}{4}+\frac{1}{5}.\]
Kovač [Ko24b] has proved that this is false - in fact as false as possible: the set of $x\in (0,\infty)$ for which the best underapproximations are eventually 'greedy' has Lebesgue measure zero. (It remains an open problem to give any explicit example of a number which is not eventually greedy, despite the fact that almost all numbers have this property.)
In [Er79] Erdős says perhaps $s_{n+1}-s_n \ll \log s_n$, but he is 'very doubtful'.
Filaseta and Trifonov [FiTr92] proved an upper bound of $s_n^{1/5}$. Pandey [Pa24] has improved this exponent to $1/5-c$ for some constant $c>0$.
The Sylvester-Gallai theorem implies that there must exist a point where only two lines from $A$ meet. This problem asks whether there must exist three such points which form a triangle (with sides induced by lines from $A$). Füredi and Palásti [FuPa84] showed this is false when $d\geq 4$ is not divisible by $9$. Escudero [Es16] showed this is false for all $d\geq 4$.
That $f(n)\to \infty$ was proved by Motzkin [Mo51]. Kelly and Moser [KeMo58] proved that $f(n)\geq\tfrac{3}{7}n$ for all $n$. This is best possible for $n=7$. Motzkin conjectured that for $n\geq 13$ there are at least $n/2$ such lines. Csima and Sawyer [CsSa93] proved a lower bound of $f(n)\geq \tfrac{6}{13}n$ when $n\geq 8$. Green and Tao [GrTa13] proved that $f(n)\geq n/2$ for sufficiently large $n$. (A proof that $f(n)\geq n/2$ for large $n$ was earlier claimed by Hansen but this proof was flawed.)
The bound of $n/2$ is best possible for even $n$, since one could take $n/2$ points on a circle and $n/2$ points at infinity. Surprisingly, Green and Tao [GrTa13] show that if $n$ is odd then $f(n)\geq 3\lfloor n/4\rfloor$.
Indeed, Shaffaf and Tao actually proved that such a rational distance set must be contained in a finite union of real algebraic curves. Solymosi and de Zeeuw [SdZ10] then proved (unconditionally) that a rational distance set contained in a real algebraic curve must be finite, unless the curve contains a line or a circle.
Ascher, Braune, and Turchet [ABT20] observed that, combined, these facts imply that a rational distance set in general position must be finite (conditional on the Bombieri-Lang conjecture).
Ascher, Braune, and Turchet [ABT20] have shown that there is a uniform upper bound on the size of such a set, conditional on the Bombieri-Lang conjecture. Greenfeld, Iliopoulou, and Peluse [GIP24] have shown (unconditionally) that any such set must be very sparse, in that if $S\subseteq [-N,N]^2$ has no three on a line and no four on a circle, and all pairwise distances integers, then \[\lvert S\rvert \ll (\log N)^{O(1)}.\]
See also [130].
In fact, such a set does exist, as proved by Jackson and Mauldin [JaMa02]. Their construction depends on the axiom of choice.
This problem is #2 in Ramsey Theory in the graphs problem collection.
In [Ru01] Ruzsa constructs an asymptotically best possible answer to this question (a so-called 'exact additive complement'); that is, there is such a set $A$ with \[\lvert A\cap\{1,\ldots,N\}\rvert \sim \frac{N}{\log_2N}\] as $N\to \infty$.
The differences are listed at A256435 on the OEIS.
Solved by Barth and Schneider [BaSc70], who proved that if $A,B\subset\mathbb{R}$ are countable dense sets then there exists a transcendental entire function $f$ such that $f(z)\in B$ if and only if $z\in A$. In [BaSc71] they proved the same result for countable dense subsets of $\mathbb{C}$.
See also [230].
For more details see the paper [BoBo09] of Bombieri and Bourgain and where Kahane's construction is improved to yield such a polynomial with \[P(z)=\sqrt{n}+O(n^{\frac{7}{18}}(\log n)^{O(1)})\] for all $z\in\mathbb{C}$ with $\lvert z\rvert=1$.
See also [228].
Erdős then asked if there is in fact an infinite string formed from $\{1,2,3,4\}$ which contains no abelian squares? This is equivalent to [192], and such a string was constructed by Keränen [Ke92]. The existence of this infinite string gives a negative answer to the problem for all $k\geq 4$.
Containing no abelian squares is a stronger property than being squarefree (the existence of infinitely long squarefree strings over alphabets with $k\geq 3$ characters was established by Thue).
We refer to a recent survey by Fici and Puzynina [FiPu23] for more background and related results.
Estimate \[m_1=\sup \overline{\delta}(A),\] where $A$ ranges over all measurable subsets of $\mathbb{R}^2$ without two points distance $1$ apart. In particular, is $m_1\leq 1/4$?
The trivial upper bound is $m_1\leq 1/2$, since for any unit vector $u$ the sets $A$ and $A+u$ must be disjoint. Erdős' question was solved by Ambrus, Csiszárik, Matolcsi, Varga, and Zsámboki [ACMVZ23] who proved that $m_1\leq 0.247$.
The values of the sum are listed at A074741 on the OEIS.
The sequence of values of $f(n)$ is A109925 on the OEIS.
See also [237].
The limit is infinite for a finite set of primes, which follows from a theorem of Pólya [Po18], that if $f(n)$ is a quadratic integer polynomial without repeated roots then as $n\to \infty$ the largest prime factor of $f(n)$ also approaches infinity. Indeed, if $P$ is a finite set of primes and $(a_i)$ is the set of integers divisible only by primes in $P$, and $a_{i+1}-a_i$ is bounded, then there exists some $k$ such that $a_{i+1}=a_i+k$ infinitely often, which contradicts Pólya's theorem with $f(n)=n(n+k)$.
Tijdeman [Ti73] proved that, if $P$ is a finite set of primes, then \[a_{i+1}-a_i \gg \frac{a_i}{(\log a_i)^C}\] for some constant $C>0$ depending on $P$.
Tijdeman [Ti73] resolved this question, proving that, for any $\epsilon>0$, there exists an infinite set of primes $P$ such that, with $a_i$ defined as above, \[a_{i+1}-a_i \gg a_i^{1-\epsilon}.\]
See also [368].
Schinzel conjectured the generalisation that, for any fixed $a$, if $n$ is sufficiently large in terms of $a$ then there exist distinct integers $1\leq x<y<z$ such that \[\frac{a}{n} = \frac{1}{x}+\frac{1}{y}+\frac{1}{z}.\]
The answer is yes, proved by Freiman [Fr73].
See also [899] for the difference set analogue.
Essentially the best possible result was proved by Tijdeman and Wagner [TiWa80], who proved that, for almost all intervals of the shape $[0,x)$, we have \[\limsup_{N\to \infty}\frac{\lvert D_N([0,x))\rvert}{\log N}\gg 1.\]
This was improved to \[f(n) \leq \exp( cn^{1/3}(\log n)^{4/3})\] by Odlyzko [Od82].
If we denote by $f^*(n)$ the analogous quantity with the assumption that $a_1<\cdots<a_n$ then Bourgain and Chang [BoCh18] prove that \[f^*(n)< \exp(c(n\log n)^{1/2}\log\log n).\]
In [Er88c] Erdős notes that Cusick had a simple proof that there do exist infinitely many such $n$. Erdős does not record what this was, but Kovač has provided the following proof: for every positive integer $m$ and $n=2^{m+1}-m-2$ we have \[\frac{n}{2^n}=\sum_{n<k\leq n+m}\frac{k}{2^k}.\]
This was essentially solved by Hančl [Ha91], who proved that such a sequence needs to satisfy \[\limsup_{n\to \infty} \frac{\log_2\log_2 a_n}{n} \geq 1.\] More generally, if $a_n\ll 2^{2^{n-F(n)}}$ with $F(n)<n$ and $\sum 2^{-F(n)}<\infty$ then $a_n$ cannot be an irrationality sequence.
Kovač and Tao [KoTa24] have proved that any strictly increasing sequence such that $\sum \frac{1}{a_n}$ converges and $\lim a_{n+1}/a_n^2=0$ is not such an irrationality sequence. On the other hand, if \[\liminf \frac{a_{n+1}}{a_n^{2+\epsilon}}>0\] for some $\epsilon>0$ then the above folklore result implies that $a_n$ is such an irrationality sequence.
Kovač and Tao [KoTa24c] have proved that $2^n$ is not such an irrationality sequence. More generally, they prove that any strictly increasing sequence of positive integers such that $\sum\frac{1}{a_n}$ converges and \[\liminf \left(a_n^2\sum_{k>n}\frac{1}{a_k^2}\right) >0 \] is not such an irrationality sequence. In particular, any strictly increasing sequence with $\limsup a_{n+1}/a_n <\infty$ is not such an irrationality sequence.
On the other hand, Kovač and Tao do prove that for any function $F$ with $\lim F(n+1)/F(n)=\infty$ there exists such an irrationality sequence with $a_n\sim F(n)$.
Erdős believed that $a_n^{1/n}\to \infty$ is possible, but $a_n^{1/2^n}\to 1$ is necessary.
This has been almost completely solved by Kovač and Tao [KoTa24], who prove that such a sequence can grow doubly exponentially. More precisely, there exists such a sequence such that $a_n^{1/\beta^n}\to \infty$ for some $\beta >1$.
It remains open whether one can achieve \[\limsup a_n^{1/2^n}>1.\] A folklore result states that $\sum \frac{1}{a_n}$ is irrational whenever $\lim a_n^{1/2^n}=\infty$, and hence such a sequence cannot grow faster than doubly exponentially - the remaining question is the precise exponent possible.
A negative answer was proved by Kovač and Tao [KoTa24], who proved even more: there exists a strictly increasing sequence of positive integers $a_n$ such that \[\sum \frac{1}{a_n+t}\] converges to a rational number for every $t\in \mathbb{Q}$ (with $t\neq -a_n$ for all $n$).
The answer is yes, proved by Kovač [Ko24], who constructs an explicit open ball inside the set. Kovač and Tao [KoTa24] have proved an analogous result for all higher dimensions.
Can the $a_k$ be explicitly determined? How fast do they grow?
Moy [Mo11] has proved that, for all such sequences, for all $\epsilon>0$, $a_k\leq (\frac{1}{2}+\epsilon)k^2$ for all sufficiently large $k$.
In general, sequences which begin with some initial segment and thereafter are continued in a greedy fashion to avoid three-term arithmetic progressions are known as Stanley sequences.
If we drop the non-empty requirement then Simonovits, Sós, and Graham [SiSoGr80] have shown that \[t\leq \binom{N}{3}+\binom{N}{2}+\binom{N}{1}+1\] and this is best possible.
Is the minimum density achieved when all the $a_i$ are equal?
For $k=1$ or $k=2$ any set $A$ such that $\sum_{n\in A}\frac{1}{n}=\infty$ has this property.
Note that since the $k$th prime is $\sim k\log k$ the lower bound $n_k>(1+\epsilon)k\log k$ is best possible here.
Is it true that for every $\epsilon>0$ there exists some $k$ such that, for every choice of congruence classes $a_i$, the density of integers not satisfying any of the congruences $a_i\pmod{n_i}$ for $1\leq i\leq k$ is less than $\epsilon$?
Does this process always terminate if $x$ has odd denominator and $A$ is the set of odd numbers? More generally, for which pairs $x$ and $A$ does this process terminate?
Graham [Gr64b] has shown that $\frac{m}{n}$ is the sum of distinct unit fractions with denominators $\equiv a\pmod{d}$ if and only if \[\left(\frac{n}{(n,(a,d))},\frac{d}{(a,d)}\right)=1.\] Does the greedy algorithm always terminate in such cases?
Graham [Gr64c] has also shown that $x$ is the sum of distinct unit fractions with square denominators if and only if $x\in [0,\pi^2/6-1)\cup [1,\pi^2/6)$. Does the greedy algorithm for this always terminate? Erdős and Graham believe not - indeed, perhaps it fails to terminate almost always.
Alekseyev [Al19] has proved this when $p(x)=x^2$, for all $m>8542$. For example, \[1=\frac{1}{2}+\frac{1}{4}+\frac{1}{6}+\frac{1}{12}\] and \[200 = 2^2+4^2+6^2+12^2.\]
This conjecture would follow for all but at most finitely many exceptions if it were known that, for all large $N$, there exists a prime $p\in [N,2N]$ such that $\frac{p+1}{2}$ is also prime.
The smallest $b$ for each $a$ are listed at A375081 at the OEIS.
This was resolved in the affirmative by van Doorn [vD24], who proved $b=b(a)$ always exists, and in fact $b(a) \ll a$. Indeed, if $a\in (3^k,3^{k+1}]$ then one can take $b=2\cdot 3^{k+1}-1$. van Doorn also proves that $b(a)>a+(1/2-o(1))\log a$, and considers various generalisations of the original problem.
It seems likely that $b(a)\leq (1+o(1))a$, and perhaps even $b(a)\leq a+(\log a)^{O(1)}$.
More generally, if the leading digit of $n$ in base $p$ is $p-1$ then $p\mid (a_n,L_n)$. There is in fact a necessary and sufficient condition: a prime $p\leq n$ divides $(a_n,L_n)$ if and only if $p$ divides the numerator of $1+\cdots+\frac{1}{k}$, where $k$ is the leading digit of $n$ in base $p$. This can be seen by writing \[a_n = \frac{L_n}{1}+\cdots+\frac{L_n}{n}\] and observing that the right-hand side is congruent to $1+\cdots+1/k$ modulo $p$. (The previous claim about $p-1$ follows immediately from Wolstenholme's theorem.)
This leads to a heuristic prediction (see for example a preprint of Shiu [Sh16]) of $\asymp\frac{x}{\log x}$ for the number of $n\in [1,x]$ such that $(a_n,L_n)=1$. In particular, there should be infinitely many $n$, but the set of such $n$ should have density zero. Unfortunately this heuristic is difficult to turn into a proof.
The answer is yes, as proved by Martin [Ma00], who in fact proved that if $B=\mathbb{N}\backslash A$ then, for all large $x$, \[\frac{\lvert B\cap [1,x]\rvert}{x}\asymp \frac{\log\log x}{\log x},\] and also gave an essentially complete description of $B$ as those integers which are small multiples of prime powers.
van Doorn has observed that if $n\in A$ (with $n>1$) then $2n\in A$ also, since if $\sum \frac{1}{m_i}=1$ then $\frac{1}{2}+\sum\frac{1}{2m_i}=1$ also.
An elementary inductive argument shows that $n_k\leq ku_k$ where $u_1=1$ and $u_{i+1}=u_i(u_i+1)$, and hence \[v(k) \leq kc_0^{2^k},\] where \[c_0=\lim_n u_n^{1/2^n}=1.26408\cdots\] is the 'Vardi constant'.
Hunter and Sawhney have observed that Theorem 3 of Bloom [Bl21] (coupled with the trivial greedy approach) implies that $k(N)=(1-o(1))\log N$.
Liu and Sawhney [LiSa24] have proved that $A(N)=(1-1/e+o(1))N$.
Estimate $f(N)$. In particular, is $f(N)=(\tfrac{1}{2}+o(1))N$?
Wouter van Doorn has given an elementary argument that proves \[f(N)\leq (25/28+o(1))N.\] Indeed, consider the sets $S_a=\{2a,3a,4a,6a,12a\}\cap [1,N]$ as $a$ ranges over all integers of the form $8^b9^cd$ with $(d,6)=1$. All such $S_a$ are disjoint and, if $A$ has no solutions to the given equation, then $A$ must omit at least two elements of $S_a$ when $a\leq N/12$ and at least one element of $S_a$ when $N/12<a\leq N/6$, and an elementary calculation concludes the proof.
Stijn Cambie and Wouter van Doorn have noted that, if we allow solutions to this equation with non-distinct $b_i$, then the size of the maximal set is at most $N/2$. Indeed, this is the classical threshold for the existence of some distinct $a,b\in A$ such that $a\mid b$.
Estimate $f(N)$. In particular, is $f(N)=(\tfrac{1}{2}+o(1))N$?
Wouter van Doorn has proved, in an unpublished note, that \[f(N) \leq (9/10+o(1))N.\] Stijn Cambie has observed that \[f(N)\geq (5/8+o(1))N,\] taking $A$ to be all odd integers $\leq N/4$ and all integers in $[N/2,N]$.
Stijn Cambie has also observed that, if we allow $b=c$, then there is a solution to this equation when $\lvert A\rvert \geq (\tfrac{2}{3}+o(1))N$, since then there must exist some $n,2n\in A$.
Related to [18].
This was solved by Yokota [Yo88], who proved that \[D(b)\ll b(\log b)(\log\log b)^4(\log\log\log b)^2.\] This was improved by Liu and Sawhney [LiSa24] to \[D(b)\ll b(\log b)(\log\log b)^3(\log\log\log b)^{O(1)}.\]
In [ErGr80] this problem is stated with the sequence $u_1=1$ and $u_{n+1}=u_n(u_n+1)$, but Quanyu Tang has pointed out this is probably an error (since with that choice we do not have $\sum \frac{1}{u_n}=1$). This question with Sylvester's sequence is the most natural interpretation of what they meant to ask.
This is not true in general, as shown by Sándor [Sa97], who observed that the proper divisors of $120$ form a counterexample. More generally, Sándor shows that for any $n\geq 2$ there exists a finite set $A\subseteq \mathbb{N}\backslash\{1\}$ with $\sum_{k\in A}\frac{1}{k}<n$ and no partition into $n$ parts each of which has $\sum_{k\in A_i}\frac{1}{k}<1$.
The minimal counterexample is $\{2,3,4,5,6,7,10,11,13,14,15\}$, found by Tom Stobart.
We may then let $A=B\cup\{1\}$ and choose $\delta(n)=-1$ for all $n\in B$ and $\delta(1)=1$.
See also [321].
See also [320].
Independently Erdős [Er36] and Chowla proved that for all $k\geq 3$ and infinitely many $n$ \[1_A^{(k)}(n) \gg n^{c/\log\log n}\] for some constant $c>0$ (depending on $k$).
For $k>2$ it is not known if $f_{k,k}(x)=o(x)$.
What if $a,b\in A$ with $a\neq b$ implies $a+b\nmid 2ab$? Must $\lvert A\rvert=o(N)$?
Wouter van Doorn has given an elementary argument that proves that if $A\subseteq \{1,\ldots,N\}$ has $\lvert A\rvert \geq (25/28+o(1))N$ then $A$ must contain $a\neq b$ with $a+b\mid ab$ (see the discussion in [301]).
See also [302].
Ruzsa suggests that a non-trivial variant of this problem arises if one imposes the stronger condition that \[\lvert A\cap \{1,\ldots,N\}\rvert \sim c_AN^{1/2}\] for some constant $c_A>0$ as $N\to \infty$, and similarly for $B$.
One can also ask what conditions are sufficient for $D(A)$ to have positive density, or for $\sum_{d\in D(A)}\frac{1}{d}=\infty$, or even just $D(A)\neq\emptyset$.
It is likely that $f(n)\leq n^{o(1)}$, or even $f(n)\leq e^{O(\sqrt{\log n})}$.
See also Problem 59 on Green's open problems list.
The set of squares has order $4$ and restricted order $5$ (see [Pa33]) and the set of triangular numbers has order $3$ and restricted order $3$ (see [Sc54]).
Is it true that if $A\backslash F$ is a basis for all finite sets $F$ then $A$ must have a restricted order? What if they are all bases of the same order?
Hegyvári, Hennecart, and Plagne [HHP07] have shown that for all $k\geq2$ there exists a basis of order $k$ which has restricted order at least \[2^{k-2}+k-1.\]
This sequence is at OEIS A005282.
See also [156].
What can be said about this sequence? Do infinitely many pairs $a,a+2$ occur? Does this sequence eventually have periodic differences? Is the density $0$?
See also Problem 7 of Green's open problems list.
The original question was answered by Szemerédi and Vu [SzVu06] (who proved that the answer is yes).
This is best possible, since Folkman [Fo66] showed that for all $\epsilon>0$ there exists a multiset $A$ with \[\lvert A\cap \{1,\ldots,N\}\rvert\ll N^{1+\epsilon}\] for all $N$, such that $A$ is not subcomplete.
This is true, and was proved by Szemerédi and Vu [SzVu06]. The stronger conjecture that this is true under \[\lvert A\cap \{1,\ldots,N\}\rvert\geq (2N)^{1/2}\] seems to be still open (this would be best possible as shown by [Er61b].
Is it true that there are infinitely many $k$ such that $T(n^k)>T(n^{k+1})$?
Kovač and Predojević [KoPr24] have proved that this is true for cyclic quadrilaterals - that is, every set with infinite measure contains four distinct points on a circle such that the quadrilateral determined by these four points has area $1$. They also prove that there exists a set of infinite measure such that every convex polygon with congruent sides and all vertices in the set has area $<1$.
Koizumi [Ko25] has resolved this question, proving that any set with infinite measure must contain the vertices of an isosceles trapezoid, an isosceles triangle, and a right-angled triangle, all of area $1$.
The original problem was solved (in the affirmative) by Beker [Be23b].
They also ask how many consecutive integers $>n$ can be represented as such a sum? Is it true that, for any $c>0$ at least $cn$ such integers are possible (for sufficiently large $n$)?
How does $f(n)$ grow? Is $f(n)=o(n)$?
If $g(n)$ is the maximal $k$ such that there are $1\leq a_1,\ldots,a_k\leq n$ with all consecutive sums distinct (i.e. we drop the monotonicity assumption in the definition of $f$) then Hegyvári [He86] has proved that \[\left(\frac{1}{3}+o(1)\right) n\leq g(n)\leq \left(\frac{2}{3}+o(1)\right)n.\]
A similar question can be asked if we replace strict monotonicity with weak monotonicity (i.e. we allow $a_i=a_j$).
Erdős and Harzheim also ask what is the least $m$ which is not a sum of the given form? Can it be much larger than $n$? Erdős and Harzheim can show that $\sum_{x<a_i<x^2}\frac{1}{a_i}\ll 1$. Is it true that $\sum_i \frac{1}{a_i}\ll 1$?
In [ErGr80] this was asked without the 'at least two' restriction, but otherwise the answer is trivially yes, as observed by Egami, since one can take $a_n=n$.
In particular, in the case $n=1$, can one prove that $a_k/k\to \infty$ and $a_k/k^{1+c}\to 0$ for any $c>0$?
See also [839].
If we further ask that $\lvert S\rvert=l$ (for any fixed $l$) then is the number of solutions \[\ll \frac{2^N}{N^2},\] with the implied constant independent of $l$ and $t$?
The second question was answered in the affirmative by Halász [Ha77], as a consequence of a more general multi-dimensional result.
This is false: Ulas [Ul05] has proved there are infinitely many solutions when $n=4$ or $n\geq 6$ and $\lvert I_i\rvert=4$ for $1\leq i\leq n$. Bauer and Bennett [BaBe07] proved there are infinitely many solutions when $n=3$ or $n=5$ and $\lvert I_i\rvert=4$ for $1\leq i\leq n$. Furthermore, Bennett and Van Luijk [BeVL12] have found infinitely many solutions when $n\geq 5$ and $\lvert I_i\rvert=5$ for $1\leq i\leq n$.
In general, Ulas conjectures there are infinitely many solutions for any fixed size of $\lvert I_i\rvert$, provided $n$ is sufficiently large.
See also [930] for a more general question.
Erdős [Er76d] believed the answer to this question is no, and in fact if $n_k$ is the $k$th powerful number then \[n_{k+2}-n_k > n_k^c\] for some constant $c>0$.
It is trivial that there are no quadruples of consecutive powerful numbers since one must be $2\pmod{4}$.
By OEIS A060355 there are no such $n$ for $n<10^{22}$.
Is the number of such $n\leq x$ bounded by $(\log x)^{O(1)}$?
The list of $n$ such that $n$ and $n+1$ are both powerful is A060355 in the OEIS.
The answer to the first question is no: Golomb [Go70] observed that both $12167=23^3$ and $12168=2^33^213^2$ are powerful. Walker [Wa76] proved that the equation \[7^3x^2=3^3y^2+1\] has infinitely many solutions, giving infinitely many counterexamples.
See also [364].
Note that $8$ is 3-full and $9$ is 2-full. Erdős and Graham asked if this is the only pair of such consecutive integers. Stephan has observed that $12167=23^3$ and $12168=2^33^213^2$ (a pair already known to Golomb [Go70]) is another example, but (by OEIS A060355) there are no other examples for $n<10^{22}$.
In [Er76d] Erdős asks the weaker question of whether there are any consecutive pairs of $3$-full integers.
The truth is probably $F(n)\gg (\log n)^2$ for all $n$. Erdős [Er76d] conjectured that, for every $\epsilon>0$, there are infinitely many $n$ such that $F(n) <(\log n)^{2+\epsilon}$.
Pasten [Pa24b] has proved that \[F(n) \gg \frac{(\log\log n)^2}{\log\log\log n}.\] The largest prime factors of $n(n+1)$ are listed as A074399 in the OEIS.
Unfortunately the problem is trivially true as written (simply taking $\{1,\ldots,k\}$ and $n>k^{1/\epsilon}$). There are (at least) two possible variants which are non-trivial, and it is not clear which Erdős and Graham meant. Let $P$ be the sequence of $k$ consecutive integers sought for. The potential strengthenings which make this non-trivial are:
See also [370].
Steinerberger has pointed out this problem has a trivial solution: take $n=m^2-1$, and then it is obvious that the largest prime factor of $n$ is $\leq m+1\ll n^{1/2}$ and the largest prime factor of $n+1$ is $\leq m\ll (n+1)^{1/2}$ (these $\ll$ can be replaced by $<$ if we choose $m$ such that $m,m+1$ are both composite).
Given that Erdős and Graham describe the above observation of Pomerance and explicitly say about this problem that 'we know very little about this', it is strange that such a trivial obstruction was overlooked. Perhaps the problem they intended was subtly different, and the problem in this form was the result of a typographical error, but I have no good guess what was intended here.
See also [369].
Hickerson conjectured the largest solution is \[16! = 14! 5!2!.\] The condition $a_1<n-1$ is necessary to rule out the trivial solutions when $n=a_2!\cdots a_k!$.
Surányi was the first to conjecture that the only non-trivial solution to $a!b!=n!$ is $6!7!=10!$.
Grimm proved that this is true if $k\ll \log n/\log\log n$. Erdős and Selfridge improved this to $k\leq (1+o(1))\log n$. Ramachandra, Shorey, and Tijdeman [RST75] have improved this to \[k\ll\left(\frac{\log n}{\log\log n}\right)^3.\]
A positive answer would imply that \[\sum_{p\leq n}1_{p\mid \binom{2n}{n}}\frac{1}{p}=(1-o(1))\log\log n,\] and Erdős, Graham, Ruzsa, and Straus say there is 'no doubt' this latter claim is true.
See also [382].
Is it true that \[Q(x)\gg_k (\log x)^k\] for every $k\geq 1$?
The answer to this problem is no: Nicolas [Ni71] proved that \[Q(x) \ll (\log x)^{O(1)}.\]
See also [380].
Tao has discussed this problem in a blog post.
Sarosh Adenwalla has observed that the first question is equivalent to [430]. Indeed, if $n$ is large and $a_i$ is the sequence defined in the latter problem, then [430] implies tha there is a composite $a_j$ such that $a_j-p(a_j)>n$ and hence $F(n)>n$.
Weisenberg has provided four easy examples that show Erdős and Graham were too optimistic here: \[\binom{7}{3}=5\cdot 7,\] \[\binom{10}{4}= 2\cdot 3\cdot 5\cdot 7,\] \[\binom{14}{4} = 7\cdot 11\cdot 13,\] and \[\binom{15}{6}=5\cdot 7\cdot 11\cdot 13.\]
Alladi and Grinstead [AlGr77] have obtained similar results when the $a_i$ are restricted to prime powers.
Solved in the affirmative by He, Juškevičius, Narayanan, and Spiro [HJNS24]. The bound of $1/n$ is the best possible, as shown by taking $z_k=1$ for $1\leq k\leq n/2$ and $z_k=i$ otherwise.
See also [498].
Pomerance [Po14] has shown that for any $k\geq 0$ there are infinitely many $n$ such that $n-k\mid\binom{2n}{n}$, although the set of such $n$ has upper density $<1/3$. Pomerance also shows that the set of $n$ such that \[\prod_{1\leq i\leq k}(n+i)\mid \binom{2n}{n}\] has density $1$.
The smallest $n$ for each $k$ are listed as A375077 on the OEIS.
Overholt [Ov93] has shown that this has only finitely many solutions assuming a weak form of the abc conjecture.
There are no other solutions below $10^9$ (see the OEIS page).
Jonas Barfield has found the solution \[10! = 48^4 - 36^4=12^4\cdot 175.\]
See also [401].
Proved for all sufficiently large sets (including the sharper version which characterises the case of equality) independently by Szegedy [Sz86] and Zaharescu [Za87].
Proved for all sets by Balasubramanian and Soundararajan [BaSo96].
See also [404].
Is there a prime $p$ and an infinite sequence $a_1<a_2<\cdots$ such that if $p^{m_k}$ is the highest power of $p$ dividing $\sum_{i\leq k}a_i!$ then $m_k\to \infty$?
Erdős and Graham ask this allowing the case $p=2$, but this is presumably an oversight, since clearly there are infinitely many solutions to this equation when $p=2$.
Brindza and Erdős [BrEr91] proved that are finitely many such solutions. Yu and Liu [YuLi96] showed that the only solutions are \[2!+1^2=3\] \[2!+5^2=3^3\] and \[4!+1^4=5^2.\]
This would imply via Kummer's theorem that \[3\mid \binom{2^{k+1}}{2^k}\] for all large $k$.
Saye [Sa22] has computed that $2^n$ contains every possible ternary digit for $16\leq n \leq 5.9\times 10^{21}$.
Erdős, Granville, Pomerance, and Spiro [EGPS90] have proved that the answer to the first two questions is yes, conditional on a form of the Elliott-Halberstam conjecture.
It is likely true that, if $k\to \infty$ however slowly with $n$, then for almost $n$ the largest prime factor of $\phi_k(n)$ is $\leq n^{o(1)}$.
The number of iterations required is A039651 in the OEIS.
Is it true that, for every $m,n\geq 2$, there exist some $i,j$ such that $\sigma_i(m)=\sigma_j(n)$?
That is, there is (eventually) only one possible sequence that the iterated sum of divisors function can settle on. Selfridge reports numerical evidence which suggests the answer is no, but Erdős and Graham write 'it seems unlikely that anything can be proved about this in the near future'.
Can one show that there exists an $\epsilon>0$ such that there are infinitely many $n$ where $m+\epsilon \omega(m)\leq n$ for all $m<n$?
Erdős also believed that $\Omega$, the count of the number of prime factors with multiplicity), should have infinitely many barriers. Selfridge found the largest barrier for $\Omega$ which is $<10^5$ is $99840$.
In [ErGr80] this problem is suggested as a way of showing that the iterated behaviour of $n\mapsto n+\omega(n)$ eventually settles into a single sequence, regardless of the starting value of $n$ (see also [412] and [414]).
Erdős and Graham report it could be attacked by sieve methods, but 'at present these methods are not strong enough'.
Weisenberg has observed that the same questions could be asked for ordering patterns which allow equality (indeed, the final problem only makes sense if we allow equality).
The behaviour of $V(x)$ is now almost completely understood. Maier and Pomerance [MaPo88] proved \[V(x)=\frac{x}{\log x}e^{(C+o(1))(\log\log\log x)^2},\] for some explicit constant $C>0$. Ford [Fo98] improved this to \[V(x)\asymp\frac{x}{\log x}e^{C_1(\log\log\log x-\log\log\log\log x)^2+C_2\log\log\log x-C_3\log\log\log\log x}\] for some explicit constants $C_1,C_2,C_3>0$. Unfortunately this falls just short of an asymptotic formula for $V(x)$ and determining whether $V(2x)/V(x)\to 2$.
In [Er79e] Erdős asks further to estimate the number of $n\leq x$ such that the smallest solution to $\phi(m)=n$ satisfies $kx<m\leq (k+1)x$.
Erdős [Er73b] has shown that a positive density set of integers cannot be written as $\sigma(n)-n$.
This is true, as shown by Browkin and Schinzel [BrSc95], who show that any integer of the shape $2^{k}\cdot 509203$ is not of this form. It seems to be open whether there is a positive density set of integers not of this form.
Mehtaab Sawhney has shared the following simple argument that proves that the above limit points are in fact the only ones.
If $v_p(m)$ is the largest $k$ such that $p^k\mid m$ then $\tau(m)=\prod_p (v_p(m)+1)$ and so \[\frac{\tau((n+1)!)}{\tau(n!)} = \prod_{p|n+1}\left(1+\frac{v_p(n+1)}{v_p(n!)+1}\right).\] Note that $v_p(n!)\geq n/p$, and furthermore $n+1$ has $<\log n$ prime divisors, each of which satisfy $v_p(n+1)<\log n$. It follows that the contribution from $p\leq n^{2/3}$ is at most \[\left(1+\frac{\log n}{n^{1/3}}\right)^{\log n}\leq 1+o(1).\]
There is at most one $p\mid n+1$ with $p\geq n^{2/3}$ which (if present) contributes exactly \[\left(1+\frac{1}{\frac{n+1}{p}}\right).\] We have proved the claim, since these two facts combined show that the ratio in question is either $1+o(1)$ or $1+1/k+o(1)$, the latter occurring if $n+1=pk$ for some $p>n^{2/3}$.
After receiving Sawhney's argument I found that this had already been proved, with essentially the same argument, by Erdős, Graham, Ivić, and Pomerance [EGIP].
In [ErGr80] (and in Guy's book) this problem as written is asking for whether almost all integers appear in this sequence, but the answer to this is trivially no (as pointed out to me by Steinerberger): no integer $\equiv 1\pmod{3}$ is ever in the sequence, so the set of integers which appear has density at most $2/3$. This is easily seen by induction, and the fact that if $a,b\in \{0,2\}\pmod{3}$ then $ab-1\in \{0,2\}\pmod{3}$.
Presumably it is the weaker question of whether a positive density of integers appear (as correctly asked in [Er77c]) that was also intended in [ErGr80].
See also Problem 63 of Green's open problems list.
If $A\subseteq \{1,\ldots,n\}$ is such that all products $a_1\cdots a_r$ are distinct for $a_1<\cdots <a_r$ then is it true that \[\lvert A\rvert \leq \pi(n)+O(n^{\frac{r+1}{2r}})?\]
Indeed, we apply this with $k=q=d$ and $a=1$ and let $p_m,\ldots,p_{m+{d-1}}$ be consecutive primes all congruent to $1$ modulo $d$, with $m>n+1$. If $p_{n+1}+\cdots+p_{m-1}\equiv r\pmod{d}$ with $1\leq r\leq d$ then \[d \mid p_{n+1}+\cdots +p_m+\cdots+p_{m+d+r-1}.\]
Is it true that, for sufficiently large $n$, not all of this sequence can be prime?
Sarosh Adenwalla has observed that this problem is equivalent to (the first part of) [385]. Indeed, assuming a positive answer to that, for all large $n$, there exists a composite $m<n$ such that all primes dividing $m$ are $>n-m$. It follows that such an $m$ is equal to some $a_i$ in the sequence defined for $[1,n)$, and $m$ is composite by assumption.
Elsholtz [El01] has proved there are no infinite sets $A,B,C$ such that $A+B+C$ agrees with the set of prime numbers up to finitely many exceptions.
See also [432].
The problem is written as Erdős and Graham describe it, but presumably they had in mind the regime where $n$ is fixed and $t\to \infty$.
A positive answer follows from work of Bui, Pratt, and Zaharescu [BPZ24], as noted by Tao in this blog post. In particular Tao shows that, if $L(x)$ is the maximal number of such squares possible, and $u(x)=(\log x\log\log x)^{1/2}$, then \[x\exp(-(2^{1/2}+o(1))u(x)) \leq L(x) \leq x\exp(-(2^{-1/2}+o(1))u(x)).\]
See also [841].
Lagarias, Odlyzko, and Shearer [LOS83] proved this is sharp for the modular version of the problem; that is, if $A\subseteq \mathbb{Z}/N\mathbb{Z}$ is such that $A+A$ contains no squares then $\lvert A\rvert\leq \tfrac{11}{32}N$. They also prove the general upper bound of $\lvert A\rvert\leq 0.475N$ for the integer problem.
In fact $\frac{11}{32}$ is sharp in general, as shown by Khalfalah, Lodha, and Szemerédi [KLS02], who proved that the maximal such $A$ satisfies $\lvert A\rvert\leq (\tfrac{11}{32}+o(1))N$.
In other words, if $G$ is the infinite graph on $\mathbb{N}$ where we connect $m,n$ by an edge if and only if $n+m$ is a square, then is the chromatic number of $G$ equal to $\aleph_0$?
This is true, as proved by Khalfalah and Szemerédi [KhSz06], who in fact prove the general result with $x+y=z^2$ replaced by $x+y=f(z)$ for any non-constant $f(z)\in \mathbb{Z}[z]$ such that $2\mid f(z)$ for some $z\in \mathbb{Z}$.
See also [438].
Is it attained by choosing all integers in $[1,(N/2)^{1/2}]$ together with all even integers in $[(N/2)^{1/2},(2N)^{1/2}]$?
If $\delta'(n)$ is the density of integers which have exactly one divisor in $(n,2n)$ then is it true that $\delta'(n)=o(\delta(n))$?
Among many other results in [Fo08], Ford also proves that the second conjecture is false, and more generally that if $\delta_r(n)$ is the density of integers with exactly $r$ divisors in $(n,2n)$ then $\delta_r(n)\gg_r\delta(n)$.
Solved by Kleitman [Kl71], who proved \[\lvert \mathcal{F}\rvert <(1+o(1))\binom{n}{\lfloor n/2\rfloor}.\]
A more precise result was proved by Hall and Tenenbaum [HaTe88] (see Section 4.6), who showed that the upper density is $\ll\epsilon \log(2/\epsilon)$. Hall and Tenenbaum further prove that $\tau^+(n)/\tau(n)$ has a distribution function.
Erdős and Graham also asked whether there is a good inequality known for $\sum_{n\leq x}\tau^+(n)$. This was provided by Ford [Fo08] who proved \[\sum_{n\leq x}\tau^+(n)\asymp x\frac{(\log x)^{1-\alpha}}{(\log\log x)^{3/2}}\] where \[\alpha=1-\frac{1+\log\log 2}{\log 2}=0.08607\cdots.\]
See also [448].
On the other hand, Cambie has observed that if $\epsilon\ll 1/n$ then $y(\epsilon,n)\sim 2n$: indeed, if $y<2n$ then this is impossible taking $x+n$ to be a multiple of the lowest common multiple of $\{n+1,\ldots,2n-1\}$. On the other hand, for every fixed $\delta\in (0,1)$ and $n$ large every $2(1+\delta)n$ consecutive elements contains many elements which are a multiple of an element in $(n,2n)$.
In [Er79d] Erdős writes that probably $n_k<e^{o(k)}$ but $n_k>k^d$ for all constant $d$.
More generally, let $q(n,k)$ denote the least prime which does not divide $\prod_{1\leq i\leq k}(n+i)$. This problem asks whether $q(n,\log n)\geq (2+\epsilon)\log n$ infinitely often. Taking $n$ to be the product of primes between $\log n$ and $(2+o(1))\log n$ gives an example where \[q(n,\log n)\geq (2+o(1))\log n.\]
Can one prove that $q(n,\log n)<(1-\epsilon)(\log n)^2$ for all large $n$ and some $\epsilon>0$?
See also [663].
This problem has consequences for [894].
Is it true that, for any $0<\delta<1/2$, we have \[N(X,\delta)=o(X)?\] In fact, is it true that (for any fixed $\delta>0$) \[N(X,\delta)<X^{1/2+o(1)}?\]
Konyagin [Ko01] proved the strong upper bound \[N(X,\delta) \ll_\delta N^{1/2}.\]
See also [466] for lower bounds.
Is there some $\delta>0$ such that \[\lim_{x\to \infty}N(X,\delta)=\infty?\]
What is the size of $D_n\backslash \cup_{m<n}D_m$?
If $f(N)$ is the minimal $n$ such that $N\in D_n$ then is it true that $f(N)=o(N)$? Perhaps just for almost all $N$?
The same question can be asked for those $n$ which do not have distinct sums of sets of divisors, but any proper divisor of $n$ does (which are listed as A119425 in the OEIS).
Benkoski and Erdős [BeEr74] ask about these two sets, and also about the set of $n$ that have a divisor expressible as a distinct sum of other divisors of $n$, but where no proper divisor of $n$ has this property.
Are there any odd weird numbers? Are there infinitely many primitive weird numbers, i.e. those such that no proper divisor of $n$ is weird?
Melfi [Me15] has proved that there are infinitely many primitive weird numbers, conditional on the fact that $p_{n+1}-p_n<\frac{1}{10}p_n^{1/2}$ for all large $n$, which in turn would follow from well-known conjectures concerning prime gaps.
The sequence of weird numbers is A006037 in the OEIS. Fang [Fa22] has shown there are no odd weird numbers below $10^{21}$, and Liddy and Riedl [LiRi18] have shown that an odd weird number must have at least 6 prime divisors.
Mrazović and Kovač, and independently Alon, have observed that the existence of some valid choice of $Q$ follows easily from Vinogradov's theorem that every large odd integer is the sum of three distinct primes. In particular, there exists some $N$ such that every prime $>N$ is the sum of three distinct (smaller) primes. We may then take $Q_0$ to be the set of all primes $\leq N$ (in which case all primes are eventually in some $Q_i$).
Watts has suggested that perhaps the obvious greedy algorithm defines such a permutation - that is, let $a_1=1$ and let \[a_{n+1}=\min \{ x : a_n+x\textrm{ is prime and }x\neq a_i\textrm{ for }i\leq n\}.\] In other words, do all positive integers occur as some such $a_n$? Do all primes occur as a sum?
This has been proved for $t\leq 12$ (see Costa and Pellegrini [CoPe20] and the references therein) and for $p-3\leq t\leq p-1$ (see Hicks, Ollis, and Schmitt [HOS19] and the references therein). Kravitz [Kr24] has proved this for \[t \leq \frac{\log p}{\log\log p}.\] (This was independently earlier observed by Will Sawin in a MathOverflow post.)
Bedert and Kravitz [BeKr24] have now proved this conjecture for \[t \leq e^{(\log p)^{1/4}}.\]
As an indication of the difficulty, when $k=3$ the smallest $n$ such that $2^n\equiv 3\pmod{n}$ is $n=4700063497$.
The minimal such $n$ for each $k$ is A036236 in the OEIS.
The original formulation of this problem had an extra condition on the minimal element of the sequence $A_k$ being large, but Ryan Alweiss has pointed out that is trivially always satisfied since the minimal element of the sequence must grow by at least $1$ at each stage.
Find similar results for $\theta=\sqrt{m}$, and other algebraic numbers.
Solved by Erdős, Sárközy, and Sós [ESS89], who in fact prove that there are at least \[\frac{N}{2}-O(N^{1-1/2^{k+1}})\] many even numbers which are of this form. They also prove that if $k=2$ then there are at least \[\frac{N}{2}-O(\log N)\] many even numbers which are of this form, and that $O(\log N)$ is best possible, since there is a $2$-colouring such that no power of $2$ is representable as a monochromatic sum.
A refinement of this problem appears as Problem 25 on the open problems list of Ben Green.
This was solved by Schinzel [Sc87], who proved that \[f(k) > \frac{\log\log k}{\log 2}.\] In fact Schinzel proves lower bounds for the corresponding problem with $P(x)^n$ for any integer $n\geq 1$, where the coefficients of the polynomial can be from any field with zero or sufficiently large positive characteristic.
Schinzel and Zannier [ScZa09] have improved this to \[f(k) \gg \log k.\]
See also [208].
This is true, and was proved by Szemerédi [Sz76].
In [Er72] Erdős goes on to ask whether \[\lim \frac{\lvert A\rvert\lvert B\rvert\log N}{N^2}\] exists, and to determine its value.
More generally, they prove that $A$ is uniquely determined by $A_k$ if $n$ is divisible by a prime greater than $k$. Selfridge and Straus sound more cautious than Erdős, and it may well be that for all $k>2$ there exist $A,B$ of the same size with identical $A_k=B_k$.
(In [Er61] Erdős states this problem incorrectly, replacing sums with products. This product formulation is easily seen to be false, as observed by Steinerberger: consider the case $k=3$ and subsets of the 6th roots of unity corresponding to $\{0,1,2,4\}$ and $\{0,2,3,4\}$ (as subsets of $\mathbb{Z}/6\mathbb{Z}$). The correct problem statement can be found in the paper of Selfridge and Straus that Erdős cites.)
This is true, and was proved by Margulis [Ma89].
Resolved by Kleitman [Kl69], who proved that the number of such families is \[2^{(1+o(1))\binom{n}{\lfloor n/2\rfloor}}.\]
See also [395].
This conjecture is true, and was proved by Marcus and Minc [MaMi62].
Erdős also conjectured the even weaker fact that there exists some $\sigma\in S_n$ such that $a_{i\sigma(i)}\neq 0$ for all $i$ and \[\sum_{i}a_{i\sigma(i)}\geq 1.\] This weaker statement was proved by Marcus and Ree [MaRe59].
van der Waerden's conjecture itself was proved by Gyires [Gy80], Egorychev [Eg81], and Falikman [Fa81].
If the sets $A_x$ are closed and have measure $<1$, then must there exist an independent set of size $3$?
Erdős writes in [Er61] that Gladysz has proved the existence of an independent set of size $2$ in the second question, but I cannot find a reference.
Hechler [He72] has shown the answer to the first question is no, assuming the continuum hypothesis.
Bannai, Bannai, and Stanton [BBS83] have proved that \[\lvert A\rvert \leq \binom{n+2}{2}.\] A simple proof of this upper bound was given by Petrov and Pohoata [PePo21].
Shengtong Zhang has observed that a simple lower bound of $\binom{n}{2}$ is given by considering all points with exactly two coordinates equal to $1$ and all others equal to $0$.
Alweiss has observed a lower bound of $\binom{n+1}{2}$ follows from considering the subset of $\mathbb{R}^{n+1}$ formed of all vectors $e_i+e_j$ where $e_i,e_j$ are distinct coordinate vectors. This set can be viewed as a subset of some $\mathbb{R}^n$, and is easily checked to have the required property.
The fact that the truth for $n=3$ is $8$ suggests that neither of these bounds is the truth.
Sendov [Se93] provided the definitive answer, proving that $\alpha_N=\pi(1-1/n)$ for $2^{n-1}+2^{n-3}<N\leq 2^n$ and $\alpha_N=\pi(1-\frac{1}{2n-1})$ for $2^{n-1}<N\leq 2^{n-1}+2^{n-3}$.
The answer is in fact no in general, as shown by Kahn and Kalai [KaKa93], who proved that it is false for $n>2014$. The current smallest $n$ where Borsuk's conjecture is known to be false is $n=64$, a result of Brouwer and Jenrich [BrJe14].
If $\alpha(n)$ is the smallest number of pieces of diameter $<1$ required (so Borsuk's original conjecture was that $\alpha(n)=n+1$) then Kahn and Kalai's construction shows that $\alpha(n)\geq (1.2)^{\sqrt{n}}$. The best upper bound, due to Schramm [Sc88], is that \[\alpha(n) \leq ((3/2)^{1/2}+o(1))^{n}.\]
This was resolved by Elliott [El67], who proved that (assuming not all points are on a circle or a line), provided $n>393$, the points determine at least $\binom{n-1}{2}$ distinct circles.
The problem appears to remain open for small $n$. Segre observed that projecting a cube onto a plane shows that the lower bound $\binom{n-1}{2}$ is false for $n=8$.
Can the length of this path be estimated in terms of $M(r)=\max_{\lvert z\rvert=r}\lvert f(z)\rvert$? Does there exist a path along which $\lvert f(z)\rvert$ tends to $\infty$ faster than a fixed function of $M(r)$ (such that $M(r)^\epsilon$)?
Solved in the affirmative by Pokrovskiy, Versteegen, and Williams [PVW24].
Wintner [Wi44] proved that, almost surely, \[\sum_{m\leq N}f(m)\ll N^{1/2+o(1)},\] and Erdős improved the right-hand side to $N^{1/2}(\log N)^{O(1)}$. Lau, Tenenbaum, and Wu [LTW13] have shown that, almost surely, \[\sum_{m\leq N}f(m)\ll N^{1/2}(\log\log N)^{2+o(1)}.\] Caich [Ca24b] has improved this to \[\sum_{m\leq N}f(m)\ll N^{1/2}(\log\log N)^{3/4+o(1)}.\] Harper [Ha13] has shown that the sum is almost surely not $O(N^{1/2}/(\log\log N)^{5/2+o(1)})$, and conjectured that in fact Erdős' conjecture is false, and almost surely \[\sum_{m\leq N}f(m) \ll N^{1/2}(\log\log N)^{1/4+o(1)}.\]
Is it true that the number of real roots of $f(z)$ is, almost surely, \[\left(\frac{\pi}{2}+o(1)\right)\log n?\]
Is it true that the number of roots of $f(z)$ in $\{ z\in \mathbb{C} : \lvert z\rvert \leq 1\}$ is, almost surely, \[\left(\frac{1}{2}+o(1)\right)n?\]
Solved by Yakir [Ya21], who proved that almost all such polynomials have \[\frac{n}{2}+O(n^{9/10})\] many roots in $\{ z\in \mathbb{C} : \lvert z\rvert \leq 1\}$.
See also [521].
Does there exist some constant $C>0$ such that, almost surely, \[\max_{\lvert z\rvert=1}\left\lvert \sum_{k\leq n}\epsilon_k(t)z^k\right\rvert=(C+o(1))\sqrt{n\log n}?\]
This was settled by Halász [Ha73], who proved this is true with $C=1$.
The answer to both questions is yes: Littlewood's conjecture was solved by Kashin [Ka87], and Konyagin [Ko94] improved this to show that $m(f)\leq n^{-1/2+o(1)}$ almost surely. This is essentially best possible, since Konyagin and Schlag [KoSc99] proved that for any $\epsilon>0$ \[\limsup_{n\to \infty} \mathbb{P}(m(f) \leq \epsilon n^{-1/2})\ll \epsilon.\] Cook and Nguyen [CoNg21] have identified the limiting distribution, proving that for any $\epsilon>0$ \[\lim_{n\to \infty} \mathbb{P}(m(f) > \epsilon n^{-1/2}) = e^{-\epsilon \lambda}\] where $\lambda$ is an explicit constant.
Kahane [Ka59] showed that $a_n=\frac{1+c}{n}$ with $c>0$ has this property, which Erdős (unpublished) improved to $a_n=\frac{1}{n}$. Erdős also showed that $a_n=\frac{1-c}{n}$ with $c>0$ does not have this property.
Solved by Shepp [Sh72], who showed that a necessary and sufficient condition is that \[\sum_n \frac{e^{a_1+\cdots+a_n}}{n^2}=\infty.\]
It is 'well known' that, for almost all $\epsilon_n=\pm 1$, the series diverges for almost all $\lvert z\rvert=1$ (assuming only $\sum \lvert a_n\rvert^2=\infty$).
Dvoretzky and Erdős [DE59] showed that if $\lvert a_n\rvert >c/\sqrt{n}$ then, for almost all $\epsilon_n=\pm 1$, the series diverges for all $\lvert z\rvert=1$.
Kesten [Ke63] proved that $C_k=2k-1-1/2k+O(1/k^2)$, and more precise asymptotics are given by Clisby, Liang, and Slade [CLS07].
Conway and Guttmann [CG93] showed that $C_2\geq 2.62$ and Alm [Al93] showed that $C_2\leq 2.696$. Jacobsen, Scullard, and Guttmann [JSG16] have computed the first few decimal places of $C_2$, showing that \[C_2 = 2.6381585303279\cdots.\]
See also [529].
For $k=2$ Duminil-Copin and Hammond [DuHa13] have proved that $d_2(n)=o(n)$.
It is now conjectured that $d_k(n)\ll n^{1/2}$ is false for $k=3$ and $k=4$, and more precisely (see for example Section 1.4 of [MaSl93]) that $d_2(n)\sim Dn^{3/4}$, $d_3(n)\sim n^{\nu}$ where $\nu\approx 0.59$, and $d_4(n)\sim D(\log n)^{1/8}n^{1/2}$.
Madras and Slade [MaSl93] have a monograph on the topic of self-avoiding walks.
See also [528].
In particular, is it true that $\ell(N)\sim N^{1/2}$?
In [AlEr85] Alon and Erdős make the stronger conjecture that perhaps $A$ can always be written as the union of at most $(1+o(1))N^{1/2}$ many Sidon sets. (This is easily verified for $A=\{1,\ldots,N\}$ using standard constructions of Sidon sets.)
Erdős and Spencer [ErSp89] proved that \[F(k) \geq 2^{ck^2/\log k}\] for some constant $c>0$. Balogh, Eberhrad, Narayanan, Treglown, and Wagner [BENTW17] have improved this to \[F(k) \geq 2^{2^{k-1}/k}.\]
They further observed that it fails for $\delta =1/4$ if we replace $K_5$ with $K_7$: by a construction of Erdős and Rogers [ErRo62] (see [620]) there exists some constant $c>0$ such that, for all large $n$, there is a graph on $n$ vertices which contains no $K_4$ and every set of at least $n^{1-c}$ vertices contains a triangle. If we take two vertex disjoint copies of this graph and add all edges between the two copies then this yields a graph on $2n$ vertices with $\geq n^2$ edges, which contains no $K_7$, yet every set of at least $2n^{1-c}$ vertices contains a triangle.
See also [579] and the entry in the graphs problem collection.
Ahlswede and Khachatrian [AhKh96] observe that it is 'easy' to find a counterexample to this conjecture, which they informed Erdős about in 1992. Erdős then gave a refined conjecture, that if $N=q_1^{k_1}\cdots q_r^{k_r}$ (where $q_1<\cdots <q_r$ are distinct primes) then the maximum is achieved by, for some $1\leq j\leq r$, those integers in $[1,N]$ which are a multiple of at least one of \[\{2q_1,\ldots,2q_j,q_1\cdots q_j\}.\] This conjecture was proved by Ahlswede and Khachatrian [AhKh96].
See also [56].
Erdős writes this is 'intimately connected' with the sunflower problem [20]. Indeed, the conjectured upper bound would follow from the following stronger version of the sunflower problem: estimate the size of the largest set of integers $A$ such that $\omega(n)=k$ for all $n\in A$ and there does not exist $a_1,\ldots,a_r\in A$ and an integer $d$ such that $(a_i,a_j)=d$ for all $i\neq j$ and $(a_i/d,d)=1$ for all $i$. The conjectured upper bound for $f_r(N)$ would follow if the size of such an $A$ must be at most $c_r^k$. The original sunflower proof of Erdős and Rado gives the upper bound $c_r^kk!$.
See also [536].
Erdős describes a construction of Ruzsa which disproves this: consider the set of all squarefree numbers of the shape $p_1\cdots p_r$ where $p_{i+1}>2p_i$ for $1\leq i<r$. This set has positive density, and hence if $A$ is its intersection with $(N/2,N)$ then $\lvert A\rvert \gg N$ for all large $N$. Suppose now that $p_1a_1=p_2a_2=p_3a_3$ where $a_i\in A$ and $p_1,p_2,p_3$ are distinct primes. Without loss of generality we may assume that $a_2>a_3$ and hence $p_2<p_3$, and so since $p_2p_3\mid a_1\in A$ we must have $2<p_3/p_2$. On the other hand $p_3/p_2=a_2/a_3\in (1,2)$, a contradiction.
Resolved by Schinzel and Szekeres [ScSz59] who proved the answer to the first question is yes and the answer to the second is no, and in fact there are examples with at most $n/(\log n)^c$ many such $m$, for some constant $c>0$.
Chen [Ch96] has proved that if $n>172509$ then \[\sum_{a\in A}\frac{1}{a}< \frac{1}{3}+\frac{1}{4}+\frac{1}{5}+\frac{1}{7}+\frac{1}{11}.\]
In [Er73] Erdős further speculates that in fact \[\sum_{a\in A}\frac{1}{a}\leq 1+o(1),\] where the $o(1)$ term $\to 0$ as $n\to \infty$.
See also [784].
This problem is #10 in Ramsey Theory in the graphs problem collection.
This problem is #11 in Ramsey Theory in the graphs problem collection.
Implied by [548].
This problem is #14 in Ramsey Theory in the graphs problem collection.
It can be easily proved by induction that every graph on $n$ vertices with at least $n(k-1)+1$ edges contains every tree on $k+1$ vertices.
Brandt and Dobson [BrDo96] have proved this for graphs of girth at least $5$. Wang, Li, and Liu [WLL00] have proved this for graphs whose complements have girth at least $5$. Saclé and Woznik [SaWo97] have proved this for graphs which contain no cycles of length $4$. Yi and Li [YiLi04] have proved this for graphs whose complements contain no cycles of length $4$.
This is false: Norin, Sun, and Zhao [NSZ16] have proved that if $T$ is the union of two stars on $k$ and $2k$ vertices, with an edge joining the centre of the two stars, then $R(T)\geq (4.2-o(1))k$. The best upper bound for the Ramsey number for this tree is $R(T)\leq 4.27492k+1$, obtained by Dubó and Stein [DuSt24].
This problem is #15 in Ramsey Theory in the graphs problem collection.
This problem is #16 in Ramsey Theory in the graphs problem collection.
This identity was proved for $k>n^2-2$ by Bondy and Erdős [BoEr73]. Nikiforov [Ni05] extended this to $k\geq 4n+2$.
Keevash, Long, and Skokan [KLS21] have proved this identity when $k\geq C\frac{\log n}{\log\log n}$ for some constant $C$, thus establishing the conjecture for sufficiently large $n$.
See also the entry in the graphs problem collection and [925].
Luczak [Lu99] has shown that $R(C_n;3)\leq (4+o(1))n$ for all $n$, and in fact $R(C_n;3)\leq 3n+o(n)$ for even $n$.
Kohayakawa, Simonovits, and Skokan [KSS05] proved this conjecture when $n$ is sufficiently large and odd. Benevides and Skokan [BeSk09] proved that if $n$ is sufficiently large and even then $R(C_n;3)=2n$.
If $G$ has $n$ vertices and maximum degree $d$ then prove that \[\hat{R}(G)\ll_d n.\]
This was disproved for $d=3$ by Rödl and Szemerédi [RoSz00], who constructed a graph on $n$ vertices with maximum degree $3$ such that \[\hat{R}(G)\gg n(\log n)^{c}\] for some absolute constant $c>0$. Tikhomirov [Ti22b] has improved this to \[\hat{R}(G)\gg n\exp(c\sqrt{\log n}).\] It is an interesting question how large $\hat{R}(G)$ can be if $G$ has maximum degree $3$. Kohayakawa, Rödl, Schacht, and Szemerédi [KRSS11] proved an upper bound of $\leq n^{5/3+o(1)}$ and Conlon, Nenadov, and Trujić [CNT22] proved $\ll n^{8/5}$. The best known upper bound of $\leq n^{3/2+o(1)}$ is due to Draganić and Petrova [DrPe22].
Determine \[\hat{R}(K_{n,n}),\] where $K_{n,n}$ is the complete bipartite graph with $n$ vertices in each component.
Conlon, Fox, and Wigderson [CFW23] have proved that, for any $s\leq t$, \[\hat{R}(K_{s,t})\gg s^{2-\frac{s}{t}}t2^s,\] and prove that when $t\gg s\log s$ we have $\hat{R}(K_{s,t})\asymp s^2t2^s$. They conjecture that this should hold for all $s\leq t$, and so in particular we should have $\hat{R}(K_{n,n})\asymp n^32^n$.
Let $F_1$ and $F_2$ be the union of stars. More precisely, let $F_1=\cup_{i\leq s} K_{1,n_i}$ and $F_2=\cup_{j\leq t} K_{1,m_j}$. Prove that \[\hat{R}(F_1,F_2) = \sum_{2\leq k\leq s+2}\max\{n_i+m_j-1 : i+j=k\}.\]
Prove that, for $r\geq 3$, \[\log_{r-1} R_r(n) \asymp_r n,\] where $\log_{r-1}$ denotes the $(r-1)$-fold iterated logarithm. That is, does $R_r(n)$ grow like \[2^{2^{\cdots n}}\] where the tower of exponentials has height $r-1$?
Prove that, for every $0\leq \alpha\leq 1/2$, \[F(n,\alpha)\sim c_\alpha\log n\] for some constant $c_\alpha$ depending only on $\alpha$.
Note that when $\alpha=0$ this is just asking for a $2$-colouring of the edges of $K_n$ which contains no monochromatic clique of size $m$, and hence we recover the classical Ramsey numbers.
See also [161].
Is it true that \[R^*(G) \leq 2^{O(n)}\] for any graph $G$ on $n$ vertices?
Rödl [Ro73] proved this when $G$ is bipartite. Kohayakawa, Prömel, and Rödl [KPR98] have proved that \[R^*(G) < 2^{O(n(\log n)^2)}.\] An alternative (and more explicit) proof was given by Fox and Sudakov [FoSu08]. Conlon, Fox, and Sudakov [CFS12] have improved this to \[R^*(G) < 2^{O(n\log n)}.\]
The graph $H_5$ can also be described as $K_4^*$, obtained from $K_4$ by subdividing one edge. ($K_4$ itself is not Ramsey size linear, since $R(4,n)\gg n^{3-o(1)}$, see [166].) Bradać, Gishboliner, and Sudakov [BGS23] have shown that every subdivision of $K_4$ on at least $6$ vertices is Ramsey size linear, and also that $R(H_5,H) \ll m$ whenever $H$ is a bipartite graph with $m$ edges and no isolated vertices.
A rational $\alpha\in [1,2]$ for which the first problem holds is known as a Turán exponent. Known Turán exponents are:
See also [713] and the entry in the graphs problem collection.
Erdős [Er64c] and Bondy and Simonovits [BoSi74] showed that \[\mathrm{ex}(n;C_{2k})\ll kn^{1+\frac{1}{k}}.\]
Benson [Be66] has proved this conjecture for $k=3$ and $k=5$. Lazebnik, Ustimenko, and Woldar [LUW95] have shown that, for arbitrary $k\geq 3$, \[\mathrm{ex}(n;C_{2k})\gg n^{1+\frac{2}{3k-3+\nu}},\] where $\nu=0$ if $k$ is odd and $\nu=1$ if $k$ is even. See [LUW99] for further history and references.
See also [765] and the entry in the graphs problem collection.
See also [574] and the entry in the graphs problem collection.
See also [573] and the entry in the graphs problem collection.
Is it true that, for every $\mathcal{F}$, if there is a bipartite graph in $\mathcal{F}$ then there exists some bipartite $G\in\mathcal{F}$ such that \[\mathrm{ex}(n;G)\ll_{\mathcal{F}}\mathrm{ex}(n;\mathcal{F})?\]
See also [180] and the entry in the graphs problem collection.
A theorem of Sudakov and Tomon [SuTo22] implies \[\mathrm{ex}(n;Q_k)=o(n^{2-\frac{1}{k}}).\] Janzer and Sudakov [JaSu22b] have improved this to \[\mathrm{ex}(n;Q_k)\ll_k n^{2-\frac{1}{k-1}+\frac{1}{(k-1)2^{k-1}}}.\] See also the entry in the graphs problem collection.
See also [533] and the entry in the graphs problem collection.
Komlós and Sós conjectured the generalisation that if at least $n/2$ vertices have degree at least $k$ then $G$ contains any tree with $k$ vertices.
Chung [Ch78] proved that every connected graph on $n$ vertices can be partitioned into at most $\lceil n/2\rceil$ edge-disjoint trees. Pyber [Py96] has shown that every connected graph on $n$ vertices can be covered by at mst $n/2+O(n^{3/4})$ paths.
If we drop the edge-disjoint condition then this conjecture was proved by Fan [Fa02].
Hajos [Lo68] has conjectured that if $G$ has all degrees even then $G$ can be partitioned into at most $\lfloor n/2\rfloor$ edge-disjoint cycles.
See also [184] and the entry in the graphs problem collection.
Fox and Sudakov [FoSu08b] have proved the second statement when $\delta >n^{-1/5}$.
Chakraborti, Janzer, Methuku, and Montgomery [CJMM24] have shown that such a graph can have at most $n(\log n)^{O(1)}$ many edges. Indeed, they prove that there exists a constant $C>0$ such that for any $k\geq 2$ there is a $c_k$ such that if a graph has $n$ vertices and at least $c_kn(\log n)^{C}$ many edges then it contains $k$ pairwise edge-disjoint cycles with the same vertex set.
Essentially solved by Nguyen and Vu [NgVu10], who proved that $\lvert A\rvert\ll N^{1/3}(\log N)^{O(1)}$.
See also [438].
This question was asked by Erdős to a young Terence Tao in 1985. We thank Tao for sharing this memory and a letter of Erdős describing the problem.
Erdős thought that $g(n) \gg n$, but in fact $g(n)=o(n)$, which follows from the density Hales-Jewett theorem proved by Furstenberg and Katznelson [FuKa91] (see [185]).
Whether this is true for $G_1=K_4$ and $G_2=K_3$ is the content of [595].
This was proved by Aharoni and Berger [AhBe09].
Erdős, Hickerson, and Pach [EHP89] proved that $u_{\sqrt{2}}(n)\asymp n^{4/3}$ and $u_D(n)\gg n\log^*n$ for all $D>1$ and $n\geq 2$ (where $\log^*$ is the iterated logarithm function).
This lower bound was improved by Swanepoel and Valtr [SwVa04] to $u_D(n) \gg n\sqrt{\log n}$. The best upper bound for general $D$ is $u_D(n)\ll n^{4/3}$.
Solved (for all sufficiently large $n$) completely by Erdős and Salamon [ErSa88]; the full description is too complicated to be given here.
Erdős and Faudree observed that every graph with $2n$ vertices and at least $n^2+1$ edges has a triangle whose vertices are joined to at least $n+2$ vertices.
Chung [Ch97] asked whether $f(n)\to \infty$ as $n\to \infty$. Day and Johnson [DaJo17] proved this is true, and that \[f(n)\geq 2^{c\sqrt{\log n}}\] for some constant $c>0$. The trivial upper bound is $2^n$.
Girão and Hunter [GiHu24] have proved that \[f(n) \ll \frac{2^n}{n^{1-o(1)}}.\]
Estimate $\tau(G)$. In particular, is it true that if $G$ has $n$ vertices then \[\tau(G) \leq n-c\sqrt{n\log n}\] for some absolute constant $c>0$?
This would be best possible, since there exist triangle-free graphs with all independent sets of size $O(\sqrt{n\log n})$, which follows from the lower bound for $R(3,k)$ by Kim [Ki95] (see [165]).
Indeed, Erdős, Gallai, and Tuza speculate that if $f(n)$ is the largest $k$ such that every triangle-free graph on $n$ vertices contains an independent set on $f(n)$ vertices, then $\tau(G)\leq n-f(n)$.
In [Er94] and [Er99] Erdős asks for a weaker upper bound $\tau(G) \leq n-\omega(n)\sqrt{n}$ for any $\omega(n)\to \infty$.
See also [611], this entry and and this entry in the graphs problem collection.
Is it true that if all maximal cliques in $G$ have at least $cn$ vertices then $\tau(G)=o_c(n)$?
Similarly, estimate for $c>0$ the minimal $k_c(n)$ such that if every maximal clique in $G$ has at least $k_c(n)$ vertices then $\tau(G)<(1-c)n$.
See also [610] and the entry in the graphs problem collection.
This was disproven for the case of $K_{2r}$-free graphs with $r\geq 2$ by Czabarka, Singgih, and Székely [CSS21], who constructed arbitrarily large connected graphs on $n$ vertices which contain no $K_{2r}$ and have minimum degree $d$, and diameter \[\frac{6r-5}{(2r-1)d+2r-3}n+O(1),\] which contradicts the above conjecture for each fixed $r$ as $d\to \infty$.
They suggest the amended conjecture, which no longer divides into two cases, that if $G$ is a connected graph on $n$ vertices with minimum degree $d$ which contains no $K_{k+1}$ then the diameter of $G$ is at most \[(3-\tfrac{2}{k})\frac{n}{d}+O(1).\] This bound is known under the weaker assumption that $G$ is $k$-colourable when $k=3$ and $k=4$, shown by Czabarka, Dankelmann, and Székely [CDS09] and Czabarka, Smith, and Székely [CSS23].
Cambie and Jooken [CaJo25] have given an example that shows the diameter for $K_4$-free graphs with minimum degree $16$ is at least $\frac{31}{216}n+O(1)$, giving another counterexample to the original conjecture.
Resolved by Fox, Loh, and Zhao [FLZ15] who showed that the answer is no; in fact they prove that \[\mathrm{rt}(n; 4, ne^{-f(n)})\geq (1/8-o(1))n^2\] whenever $f(n) =o(\sqrt{\log n/\log\log n})$.
See also [22] and the entry in the graphs problem collection.
Determine the best possible $t$ such that, if $G$ is an $r$-uniform hypergraph $G$ where every subgraph $G'$ on at most $3r-3$ vertices has $\tau(G')\leq 1$, we have $\tau(G)\leq t$.
Erdős, Gyárfás, and Ruszinkó [EGR98] proved that if $G$ has no isolated vertices and maximum degree $O(1)$ then $h_2(G)\ll n\log n$.
Alon has observed this problem is essentially identical to [134], and his solution in this note also solves this problem in the affirmative.
See also [619].
Is it true that there exists a constant $c>0$ such that if $G$ is a connected graph on $n$ vertices then $h_4(G)<(1-c)n$?
If we omit the condition that the graph must remain triangle-free then Alon, Gyárfás, and Ruszinkó [AGR00] have proved that adding $n/2$ edges always suffices to obtain diameter at most $4$.
It is now known that $f(n)=n^{1/2+o(1)}$. Bollobás and Hind [BoHi91] proved \[n^{1/2} \ll f(n) \ll n^{7/10+o(1)}.\] Krivelevich [Kr94] improved this to \[n^{1/2}(\log\log n)^{1/2} \ll f(n) \ll n^{2/3}(\log n)^{1/3}.\] Wolfovitz [Wo13] proved \[f(n) \ll n^{1/2}(\log n)^{120}.\]
Is it true that \[\alpha_1(G)+\tau_1(G) \leq \frac{n^2}{4}?\]
For this weaker statement, Erdős and Gyárfás conjectured the stronger form that if $\lvert X\rvert=2^k$ then, for any $f:\{A : A\subseteq X\}\to X$, there must exist some $Y\subset X$ of size $k$ such that \[\#\{ f(A) : A\subseteq Y\}< 2^k-k^C\] for every $C$ (with $k$ sufficiently large depending on $C$). This was proved by Alon (personal communication), who proved the stronger version that, for any $\epsilon>0$, if $k$ is large enough, there must exist some $Y$ of size $k$ such that \[\#\{ f(A) : A\subseteq Y\}<(1-\epsilon)2^k.\] Alon also proved that, provided $k$ is large enough, if $\lvert X\rvert=2^k$ there exists some $f:\{A: A\subseteq X\}\to X$ such that, if $Y\subset X$ with $\lvert Y\rvert=k$, then \[\#\{ f(A) : A\subseteq Y\}>\tfrac{1}{4}2^k.\]
Conversely, if $h^{(m)}(n)$ is the maximal chromatic number of a graph on $n$ vertices with girth $>m$ then does \[\lim_{n\to \infty}\frac{\log h^{(m)}(n)}{\log n}\] exist, and what is its value?
Erdős [Er59b] proved that \[\lim_{n\to \infty}\frac{\log h^{(m)}(n)}{\log n}\gg \frac{1}{m}\] and, for odd $m$, \[\lim_{n\to \infty}\frac{\log h^{(m)}(n)}{\log n}\leq \frac{2}{m+1},\] and conjectured this is sharp. He had no good guess for the value of the limit for even $m$, other that it should lie in $[\frac{2}{m+2},\frac{2}{m}]$, but could not prove this even for $m=4$.
Erdős [Er67c] proved that \[f(n) \asymp \frac{n}{(\log n)^2}\] and that the limit in question, if it exists, must be in \[(\log 2)^2\cdot [1/4,1].\]
Balogh, Kostochka, Prince, and Stiebitz [BKPS09] have proved the full conjecture for quasi-line graphs and graphs with independence number $2$.
Determine the minimal number of vertices $n(k)$ of a bipartite graph $G$ such that $\chi_L(G)>k$.
Erdős, Rubin, and Taylor [ERT80] proved $n(2)=6$ and Hanson, MacGillivray, and Toft [HMT96] proved $n(3)=14$ and \[n(k) \leq kn(k-2)+2^k.\]
Does every planar bipartite graph $G$ have $\chi_L(G)\leq 3$?
Does every planar graph $G$ have $\chi_L(G)\leq 5$? Is this best possible?
If $G$ is $(a,b)$-choosable then $G$ is $(am,bm)$-choosable for every integer $m\geq 1$.
This is false: Dvořák, Hu, and Sereni [DHS19] construct a graph which is $(4,1)$-choosable but not $(8,2)$-choosable.
This was proved by Kwan and Sudakov [KwSu21].
This was proved by Bukh and Sudakov [BuSu07].
Jenssen, Keevash, Long, and Yepremyan [JKLY20] have proved that there must exist an induced subgraph which contains $\gg n^{2/3}$ distinct degrees (with no restriction on the number of vertices).
Is it true that for every infinite cardinal $\aleph$ there is a graph $G$ of which every finite subgraph is in $S$ and if the edges of $G$ are coloured with $\aleph$ many colours then there is a monochromatic triangle.
This problem was solved completely by Keevash and Sudakov [KeSu04], who provd that the corret threshold is $\lfloor n^2/4\rfloor$ for all $n\geq 7$, is $\binom{n}{2}$ for $n\leq 5$, and is $10$ for $n=6$.
This was resolved in the negative by Janzer, Steiner, and Sudakov [JSS24] - in fact, this fails even at $k=2$. Janzer, Steiner, and Sudakov proved that there exists a constant $c>0$ such that, for all large $n$, there exists a graph on $n$ vertices with chromatic number \[\geq c\frac{\log\log n}{\log\log \log n}\] which contains no $4$-regular subgraph.
Füredi proved that $f(n;3) \ll n^2$ and $f(n;3) > \binom{n}{2}$ for infinitely many $n$. More generally, Füredi proved that \[f(n;t) \ll \binom{n}{t-1}.\]
Ryan Alweiss has provided the following simple argument showing that the answer is yes: suppose we have some red/blue colouring without this property. Without loss of generality, suppose $1$ is coloured red, and then either $3$ or $5$ must be blue.
Suppose first that $3$ is blue. If $n\geq 6$ is red then (considering $1,n,2n-1$) we deduce $2n-1$ is blue, and then (considering $3,n+1,2n-1$) we deduce that $n+1$ is red. In particular the colouring must be eventually constant, and we are done.
Now suppose that $5$ is blue. Arguing similarly (considering $1,n,2n-1$ and $5,n+2,2n-1$) we deduce that if $n\geq 8$ is red then $n+2$ is also red, and we are similarly done, since the colouring must be eventually constant on some congruence class modulo $2$.
In [Er79] Erdős says 'it is extremely doubtful' that there are infinitely many such $n$, and in fact suggets that \[\lim_{n\to \infty}\max_{m<n}(\tau(m)+m-n)=\infty.\]
In [Er79d] Erdős says it 'seems certain' that for every $k$ there are infinitely many $n$ for which \[\max_{n-k<m<n}(m+\tau(m))\leq n+2,\] but 'this is hopeless with our present methods', although it follows from Schinzel's Hypothesis H.
See also [413].
In fact, the answer to this question as written is easily seen to be no, since there are no solutions to $2^k\equiv -1\pmod{7}$, and hence this fails with $p=2$ and $q=7$. It is possible that Erdős meant to exclude such obstructions, by amending this to 'odd primes' or 'all sufficiently large primes' or such.
Even with such amendments, this problem is false is a strong sense: Alan Tong has provided the following elegant elementary proof that, for any given prime $p$, there are infinitely many primes $q$ such that this statement is false: let $m$ be the product of all primes $\leq p$, and choose a prime $q$ congruent to $-1$ modulo $4m$. If $p$ is the greatest prime divisor of $n$ then, using quadratic reciprocity, every prime divisor of $n$ is a quadratic residue modulo $q$, and hence $n$ is a quadratic residue modulo $q$. On the other hand, since $q\equiv 3\pmod{4}$ we know that $-1$ is not a quadratic residue modulo $q$, and hence $n\not\equiv -1\pmod{q}$, so it is impossible for $q\mid n+1$.
Tong asks whether, for any given odd prime $q$, there are infinitely many primes $p$ such that there is no integer $n$ with $P(n)=p$ and $P(n+1)=q$.
Sampaio independently observed that the answer to Erdős' original problem is no if one of the primes can be $2$ - for example this is false with $p=19$ and $q=2$, since if $n+1=2^k$ and $19\mid n$ then (since $2$ is a primitive root modulo $19$) we must have $18\mid k$, and hence $73\mid 2^{18}-1\mid n$. A similar argument works with $19$ replaced by any prime $p>13$ for which $2$ is a primitive root, using a result of Rotkiewicz [Ro64b] that for every prime $p>13$ there is a prime $q>p$ which divides $2^{p-1}-1$.
Problem 6 in the 12th Romanian Master of Mathematics Competitions in 2020 was to prove that there exist infinitely many odd primes $p$ such that, for every $n$, $P(n)P(n+1)\neq 2p$.
Estimate $f(m)$. In particular is it true that $f(m)\ll m^{1/2}$?
Zach Hunter has observed that taking $n$ points equally spaced on a circle disproves this conjecture. In the spirit of related conjectures of Erdős and others, presumably some kind of assumption that the points are in general position (e.g. no three on a line and no four on a circle) was intended.
More generally, one can ask how many distances $A$ must determine if every set of $p$ points determines at least $q$ points.
See also [657].
In [Er73] Erdős says it is not even known in $\mathbb{R}$ whether $f(n)\to \infty$. Sarosh Adenwalla has observed that this is equivalent to minimising the number of distinct differences in a set $A\subset \mathbb{R}$ of size $n$ without three-term arithmetic progressions. Dumitrescu [Du08] proved that, in these terms, \[(\log n)^c \leq f(n) \leq 2^{O(\sqrt{\log n})}\] for some constant $c>0$.
Straus has observed that if $2^k\geq n$ then there exist $n$ points in $\mathbb{R}^k$ which contain no isosceles triangle and determine at most $n-1$ distances.
See also [656].
This qualitative statement follows from the density Hales-Jewett theorem proved by Furstenberg and Katznelson [FuKa91]. A quantitative proof (yet with very poor bounds) was given by Solymosi [So04].
More generally, if $F(2n)$ is the minimal number of such distances, and $f(2n)$ is minimal number of distinct distances between any $2n$ points in $\mathbb{R}^2$, then is $f \ll F$?
See also [89].
Let $x_1,\ldots,x_n\in \mathbb{R}^2$ be such that $d(x_i,x_j)\geq 1$ for all $i\neq j$. Is it true that, provided $n$ is sufficiently large depending on $t$, the number of distances $d(x_i,x_j)\leq t$ is less than or equal to $f(t)$ with equality perhaps only for the triangular lattice?
In particular, is it true that the number of distances $\leq \sqrt{3}-\epsilon$ is less than $1$?
This is essentially verbatim the problem description in [Er97e], but this does not make sense as written; there must be at least one typo. Suggestions about what this problem intends are welcome.
Erdős also goes on to write 'Perhaps the following stronger conjecture holds: Let $t_1<t_2<\cdots$ be the set of distances occurring in the triangular lattice. $t_1=1$ $t_2=\sqrt{3}$ $t_3=3$ $t_4=5$ etc. Is it true that there is an $\epsilon_n$ so that for every set $y_1,\ldots,$ with $d(y_i,y_j)\geq 1$ the number of distances $d(y_i,y_j)<t_n$ is less than $f(t_n)$?'
Again, this is nonsense interpreted literally; I am not sure what Erdős intended.
The bound $q(n,k)<(1+o(1))k\log n$ is easy. It may be true this improved bound holds even up to $k=o(\log n)$.
See also [457].
Must there exist some set $B$ such that $B\cap A_i\neq \emptyset$ and $\lvert B\cap A_i\rvert \ll_c 1$ for all $i$?
In [Er81] the condition $\lvert A_i\cap A_j\rvert\leq 1$ for all $i\neq j$ is replaced by every two points in $\{1,\ldots,n\}$ being contained in exactly one $A_i$, that is, $A_1,\ldots,A_m$ is a pairwise balanced block design (and the condition $c<1$ is omitted).
Alon has proved that the answer is no: if $q$ is a large prime power and $n=m=q^2+q+1$ then there exist $A_1,\ldots,A_m\subseteq \{1,\ldots,n\}$ such that $\lvert A_i\rvert \geq \tfrac{2}{5}\sqrt{n}$ for all $i$ and $\lvert A_i\cap A_j\rvert\leq 1$ for all $i\neq j$, and yet if $B$ has non-empty intersection with all $A_i$ then there exists $A_j$ such that $\lvert B\cap A_j\rvert \gg \log n$. (The construction is to take random subsets of the lines of a projective plane.)
The weaker version that Erdős posed in [Er81] remains open, although Alon conjectures the answer there to also be no.
Shrikhande and Singhi [ShSi85] have proved that the answer is no conditional on the conjecture that the order of every projective plane is a prime power (see [723]), by proving that every pairwise balanced design on $n$ points in which each block is of size $\geq n^{1/2}-c$ can be embedded in a projective plane of order $n+i$ for some $i\leq c+2$, if $n$ is sufficiently large.
Erdős asks if this is false for constant, for which functions $h(n)$ will the condition $\lvert A_i\rvert \geq n^{1/2}-h(n)$ make the conjecture true?
The answer to this problem is no: Chung [Ch92] and Brouwer, Dejter, and Thomassen [BDT93] constructed an edge-partition of $Q_n$ into four subgraphs, each containing no $C_6$.
See also [86].
When $q=1$ this corresponds exactly to the classical Ramsey problem, and hence for example \[\frac{1}{p-1}\leq c(p,1) \leq \frac{2}{p+1}.\] It is easy to see that if $q=\binom{p-1}{2}+1$ then $c(p,q)=1$. Erdős, Faudree, Rousseau, and Schelp have shown that $c(p,\binom{p-1}{2})\leq 1/2$.
Estimate $f_k(n)$ and $F_k(n)$ - in particular, determine $\lim F_k(n)/n^2$ and $\lim f_k(n)/n^2$.
The theory of Pell equations implies that there are infinitely many pairs $n,d$ with $(n,d)=1$ such that $n(n+d)(n+2d)$ is a square.
Considering the question of whether the product of an arithmetic progression of length $k$ can be equal to an $\ell$th power:
Jakob Führer has observed this is possible for integers in general, for example $(-6)\cdot(-1)\cdot 4\cdot 9=6^3$.Terence Tao has observed that, for any divisor $m\mid n$, \[\frac{\tau(n/m)}{m} \leq G(n) \leq \tau(n),\] and hence for example $\tau(n)/4\leq G(n)\leq \tau(n)$ for even $n$. It is easy to then see that $G(n)$ grows on average, and in general behaves very similarly to $\tau(n)$ (and in particular the answer to the first question is yes). Tao suggests that this was a mistaken conjecture of Erdős, which he soon corrected a year later to [448].
Indeed, in [Er82e] Erdős recalls this conjecture and observes that it is indeed trivial that $G(n)\to \infty$ for almost all $n$, and notes that he and Tenenbaum proved that $G(n)/\tau(n)$ has a continuous distribution function.
More generally, Brun's sieve can be used to prove that if $B\subseteq \mathbb{N}$ is a set of pairwise coprime integers with $\sum_{b<x}\frac{1}{b}=o(\log\log x)$ then $A=\{ n: b\nmid n\textrm{ for all }b\in A\}$ has the translation property. Erdős did not know what happens if the condition on $\sum_{b<x}\frac{1}{b}$ is weakened or dropped altogether.
What if the condition that $p$ is prime is omitted? Selfridge and Wagstaff made a 'preliminary computer search' and suggested that there are infinitely many $n$ not of this form even without the condition that $p$ is prime. It should be true that the number of exceptions in $[1,x]$ is $<x^c$ for some constant $c<1$.
Most generally, given some infinite set $A\subseteq \mathbb{N}$ and function $f:A\to \mathbb{N}$ one can ask for sufficient conditions on $A$ and $f$ that guarantee every large number (or almost all numbers) can be written as \[am^2+b\] for some $m\in A$ and $a\geq 1$ and $0\leq b<f(m)$.
In another direction, one can ask what is the minimal $c_n$ such that $n$ can be written as $n=ap^2+b$ with $0\leq b<c_np$ for some $p\leq \sqrt{n}$. This problem asks whether $c_n\leq 1$ eventually, but in [Er79d] Erdős suggests that in fact $\limsup c_n=\infty$. Is it true that $c_n<n^{o(1)}$?
Is it true that for all $m\geq n+k$ \[M(n,k) \neq M(m,k)?\]
In general, how many solutions does $M(n,k)=M(m,l)$ have when $m\geq n+k$ and $l>1$? Erdős expects very few (and none when $l\geq k$).
The only solutions Erdős knew were $M(4,3)=M(13,2)$ and $M(3,4)=M(19,2)$.
In [Er79d] Erdős conjectures the stronger fact that (aside from a finite number of exceptions) if $k>2$ and $m\geq n+k$ then $\prod_{i\leq k}(n+i)$ and $\prod_{i\leq k}(m+i)$ cannot have the same set of prime factors.
Let $k\geq 3$. Are there infinitely many $m,n$ with $m\geq n+k$ such that \[M(n,k)>M(m,k+1)?\]
The answer is yes, as proved in a strong form by Cambie [Ca24].
It is easy to see that there are infinitely many solutions to $M(n,k)>M(m,k)$. If $n_k$ is the smallest $n$ with this property (for some $m$) then are there good bounds for $n_k$? Erdős writes that he could prove $n_k/k\to \infty$, but knew of no good upper bounds.
Erdős also asked the following: If $u_k$ is minimal such that $M(u_k,k)>M(u_k+1,k)$ and $t<\min(u_k,T)$ then is it true that $M(t,k)\leq M(T,k)$? Stijn Cambie and Wouter van Doorn have observed that there are many counterexamples to this with $t=u_k-1$ and $T=u_k+1$. For example, when $k=7$ we have $u_k=7$, yet $M(6,7)=M(7,7)>M(8,7)$.
See also [677].
Can one show the stronger version with \[\omega(n-k) < \frac{\log k}{\log\log k}+O(1)\] is false?
Can one prove this is false if we replace $k^2+1$ by $e^{(1+\epsilon)\sqrt{k}}+C_\epsilon$, for all $\epsilon>0$, where $C_\epsilon>0$ is some constant?
Erdős observed that Cramer's conjecture \[\limsup_{k\to \infty} \frac{p_{k+1}-p_k}{(\log k)^2}=1\] implies that for all $\epsilon>0$ and all sufficiently large $n$ there exists some $k$ such that \[p(n+k)>e^{(1-\epsilon)\sqrt{k}}.\] There is now evidence, however, that Cramer's conjecture is false; a more refined heuristic by Granville [Gr95] suggests this $\limsup$ is $2e^{-\gamma}\approx 1.119\cdots$, and so perhaps the $1+\epsilon$ in the second question should be replaced by $2e^{-\gamma}+\epsilon$.
Erdős [Er79d] writes it 'seems certain' that this holds for every $c>0$, with only a finite number of exceptions (depending on $c$). Standard heuristics on prime gaps suggest that the largest prime divisor of $\binom{n}{k}$ is in fact \[> \min(n-k+1, e^{c\sqrt{k}})\] for some constant $c>0$.
Let $f(n)$ be the smallest $k$ such that $u>n^2$. Give bounds for $f(n)$.
Mahler's theorem implies $f(n)\to \infty$ as $n\to \infty$, but is ineffective, and so gives no bounds on the growth of $f(n)$.
One can similarly ask for estimates on the smallest integer $f(n,k)$ such that if $m$ is the factor of $\binom{n}{k}$ containing all primes $\leq f(n,k)$ then $m > n^2$.
Give good estimates for $Y(x)$. In particular, can one prove that $Y(x)=o(x^2)$ or even $Y(x)\ll x^{1+o(1)}$?
Maier and Pomerance have conjectured that $Y(x)\ll x(\log x)^{2+o(1)}$.
Estimate $\epsilon_n$ - in particular is it true that $\epsilon_n=o(1)$?
For fixed $k\geq 1$ is $d_k(p)$ unimodular in $p$? That is, it first increases in $p$ until its maximum then decreases.
A similar question can be asked if we consider the density of integers whose $k$th smallest divisor is $d$. Erdős could show that this function is not unimodular.
Cambie [Ca25] has shown that $d_k(p)$ is unimodular for $1\leq k\leq 3$ and is not unimodular for $4\leq k\leq 20$.
The general situation is more complicated. For example suppose $A$ is the union of $(n_k,(1+\eta_k)n_k)\cap \mathbb{Z}$ where $1\leq n_1<n_2<\cdots$ is a lacunary sequence. If $\sum \eta_k<\infty$ then the density of $M_A$ exists and is $<1$. If $\eta_k=1/k$, so $\sum \eta_k=\infty$, then the density exists and is $<1$.
Erdős writes it 'seems certain' that there is some threshold $\alpha\in (0,1)$ such that, if $\eta_k=k^{-\beta}$, then the density of $M_A$ is $1$ if $\beta <\alpha$ and the density is $<1$ if $\beta >\alpha$.
Cambie has calculated that unimodularity fails even for $n=2$ and $n=3$. For example, \[\delta_1(3,6)= 0.35\quad \delta_1(3,7)\approx 0.33\quad \delta_1(3,8)\approx 0.3619.\]
Furthermore, Cambie [Ca25] has shown that, for fixed $n$, the sequence $\delta_1(n,m)$ has superpolynomially many local maxima $m$.
See also [446].
See also [51].
See also [696].
Estimate $h(n)$ and $H(n)$. Is it true that $H(n)/h(n)\to \infty$ for almost all $n$?
See also [695].
See also [696].
Erdős and Szekeres further conjectured that $p\geq i$ can be improved to $p>i$ except in a few special cases. In particular this fails when $i=2$ and $n$ being some particular powers of $2$. They also found some counterexamples when $i=3$, but only one counterexample when $i\geq 4$: \[\textrm{gcd}\left(\binom{28}{5},\binom{28}{14}\right)=2^3\cdot 3^3\cdot 5.\]
It is known that $f(n)=n/P(n)$ when $n$ is the product of two primes. Another example is $n=30$.
For the second problem, it is easy to see that for any $n$ we have $f(n)\geq p(n)$, where $p(n)$ is the smallest prime dividing $n$, and hence there are infinitely many $n$ (those $=p^2)$ such that $f(n)\geq n^{1/2}$.
Sterboul [St74] proved this when, letting $\mathcal{G}$ be the maximal sets (under inclusion) in $\mathcal{F}$, all sets in $\mathcal{G}$ have the same size, $\lvert A\cap B\rvert\leq 1$ for all $A\neq B\in \mathcal{G}$, and at least two sets in $\mathcal{G}$ have non-empty intersection.
Frankl and Kupavskii [FrKu23] have proved this when $\mathcal{F}$ has covering number $2$.
Borg [Bo11] has proposed a weighted generalisation of this conjecture, which he proves under certain additional assumptions.
Estimate the chromatic number $\chi(G_n)$. Does it grow exponentially in $n$? Does \[\lim_{n\to \infty}\chi(G_n)^{1/n}\] exist?
Is there some $k$ such that if $G$ has girth $\geq k$ (i.e. $G$ contains no cycles of length $<k$) then $\chi(G)\leq 3$?
Wormald [Wo79] has constructed a unit distance graph with $\chi(G)=4$ and girth $5$, with $6448$ vertices. O'Donnell [OD94] has constructed a unit distance graph with $\chi(G)=4$ and girth $4$, with $56$ vertices. Chilakamarri [Ch95] has constructed an infinite family of unit distance graphs with $\chi(G)=4$ and girth $4$, the smallest of which has $47$ vertices.
Estimate $L(r)$. In particular, is it true that $L(r)\leq r^{O(1)}$?
Obtain good bounds for $f(n)$, or even an asymptotic formula.
The answer is yes, proved by Tashkinov [Ta82].
In [Er81] Erdős asks whether the same is true for any $3$-uniform hypergraph on $k$ vertices with $k-3$ $3$-edges.
The answer is yes, proved by Fox, Lee, and Sudakov [FLS13].
Mader [Ma67] proved that $\geq 2^{\binom{r}{2}}n$ edges suffices.
The answer is yes, proved independently by Komlós and Szemerédi [KoSz96] and Bollobás and Thomason [BoTh96].
Is every $r$-hypergraph $G$ on $n$ vertices the union of at most $\mathrm{ex}_{r}(n;K_{r+1}^r)$ many copies of $K_r^r$ and $K_{r+1}^r$, no two of which share a $K_r^r$?
Give reasonable bounds for $W(3,k)$. In particular, give any non-trivial lower bounds for $W(3,k)$ and prove that $W(3,k) < \exp(k^c)$ for some constant $c<1$.
Green [Gr22] established the superpolynomial lower bound \[W(3,k) \geq \exp\left( c\frac{(\log k)^{4/3}}{(\log\log k) ^{1/3}}\right)\] for some constant $c>0$ (in particular disproving a conjecture of Graham that $W(3,k)\ll k^2$). Hunter [Hu22] improved this to \[W(3,k) \geq \exp\left( c\frac{(\log k)^{2}}{\log\log k}\right).\] The first to show that $W(3,k) < \exp(k^c)$ for some $c<1$ was Schoen [Sc21]. The best upper bound currently known is \[W(3,k) \ll \exp\left( O((\log k)^9)\right),\] which follows from the best bounds known for sets without three-term arithmetic progressions (see [BlSi23] which improves slightly on the bounds due to Kelley and Meka [KeMe23]).
That is, can one find a family of $\binom{n}{k}\binom{k}{r}^{-1}$ many subsets of $\{1,\ldots,n\}$, all of size $k$, such that any $A\subseteq \{1,\ldots,n\}$ of size $r$ is contained in exactly one set in the family?
A finite projective plane of order $n$ is a collection of subsets of $\{1,\ldots,n^2+n+1\}$ of size $n+1$ such that every pair of elements is contained in exactly one set.
Bruck and Ryser [BrRy49] have proved that if $n\equiv 1\pmod{4}$ or $n\equiv 2\pmod{4}$ then $n$ must be the sum of two squares. For example, this rules out $n=6$ or $n=14$. The case $n=10$ was ruled out by computer search [La97].
Chowla, Erdős, and Straus [CES60] proved $f(n) \gg n^{1/91}$. Wilson [Wi74] proved $f(n) \gg n^{1/17}$. Beth [Be83c] proved $f(n) \gg n^{1/14.8}$.
The sequence of $f(n)$ is A001438 in the OEIS.
Balakran [Ba29] proved this holds for $k=1$ - that is, $(n+1)^2\mid \binom{2n}{n}$ for infinitely many $n$. It is a classical fact that $(n+1)\mid \binom{2n}{n}$ for all $n$ (see Catalan numbers).
Erdős, Graham, Ruzsa, and Straus observe that the method of Balakran can be further used to prove that there are infinitely many $n$ such that \[(n+k)!(n+1)! \mid (2n)!\] (in fact this holds whenever $k<c \log n$ for some small constant $c>0$).
Erdős [Er68c] proved that if $a!b!\mid n!$ then $a+b\leq n+O(\log n)$.
By Legendre's formula $a! b! \mid n!(a+b-n)!$ is true if and only if for all primes $p$ \[s_p(n)+s_p(a+b-n) \leq s_p(a)+s_p(b),\] where $s_p(n)$ is the sum of the base $p$ digits of $n$.
See also [729].
This problem is asking if $a!b!\mid n!$ 'ignoring what happens on small primes' still implies $a+b+\leq n+O(\log n)$.
See also [728].
For example $(87,88)$ and $(607,608)$.
Kummer's theorem implies that, for all odd primes $p$, $p\mid \binom{2n}{n}$ if and only some base $p$ digit of $n$ is $>p/2$, and hence $(n,n+1)$ has the required property if for all primes $p\leq n$ we have $n\not\in \{\frac{p-1}{2},p-1\}\pmod{p}$. Standard heuristics then predict there should be \[\gg \frac{x}{(\log x)^2}\] many such $n\leq x$.
Are there necessary and sufficient conditions for $(X_i)$ to be block-compatible?
Is there some constant $c>0$ such that for all large $n$ there are \[\geq \exp(c n^{1/2}\log n)\] many block-compatible sequences for $\{1,\ldots,n\}$?
He could prove that there are \[\leq \exp(O(n^{1/2}\log n))\] many block-compatible sequences for $\{1,\ldots,n\}$.
Alon has proved there are at least \[2^{(\frac{1}{2}+o(1))n^{1/2}\log n}\]
many sequences which are block-compatible for $n$. See also [733].
Prove that there are at most \[\exp(O(n^{1/2}))\] many line-compatible sequences.
Erdős writes that it is 'easy' to prove there are at least \[\exp(cn^{1/2})\] many such sequences for some constant $c>0$, but expected proving the upper bound to be difficult. Once it is done, he asked for the existence and value of \[\lim_{n\to \infty}\frac{\log f(n)}{n^{1/2}},\] where $f(n)$ counts the number of line-compatible sequences.
This is true, and was proved by Szemerédi and Trotter [SzTr83].
See also [732].
Erdős [Er81] writes 'this will be probably not be very difficult to prove but so far I was not successful'.
Erdős and de Bruijn [dBEr48] proved that if $A_1,\ldots,A_m\subseteq \{1,\ldots,n\}$ is a pairwise balanced block design then $m\geq n$, and this implies there must be some $t$ such that there are $\gg n^{1/2}$ many $t$ with $\lvert A_i\rvert=t$.
The previous configurations are the only examples, as proved by Ackerman, Buchin, Knauer, Pinchasi, and Rote [ABKPR08].
More generally, Erdős asks to characterise families $\mathcal{F}_\alpha$ of finite graphs such that there is a graph of chromatic number $\aleph_\alpha$ all of whose finite subgraphs are in $\mathcal{F}_\alpha$.
More generally, Erdős and Hajnal asked must there exist (for every cardinal $\mathfrak{m}$ and integer $r$) some $f_r(\mathfrak{m})$ such that every graph with chromatic number $\geq f_r(\mathfrak{m})$ contains a subgraph with chromatic number $\mathfrak{m}$ with no odd cycle of length $\leq r$?
Erdős [Er95d] claimed that even the $r=3$ case of this is open: must every graph with sufficiently large chromatic number contain a triangle free graph with chromatic number $\mathfrak{m}$?
In [Er81] Erdős also asks the same question but with girth (i.e. the subgraph does not contain any cycle at all of length $\leq C$).
Is there a basis $A$ of order $2$ such that if $A=A_1\sqcup A_2$ then $A_1+A_1$ and $A_2+A_2$ cannot both have bounded gaps?
This is true (for sufficiently large $n$) and was proved by Füredi [Fu92].
Gyárfás and Lehel [GyLe78] proved that this holds if all but at most $2$ of the trees are stars, or if all the trees are stars or paths. Fishburn [Fi83] proved this for $n\leq 9$. Bollobás [Bo83] proved that the smallest $\lfloor n/\sqrt{2}\rfloor$ many trees can always be packed greedily into $K_n$.
Joos, Kim, Kühn, and Osthus [JKKO19] proved that this conjecture holds when the trees have bounded maximum degree. Allen, Böttcher, Clemens, Hladky, Piguet, and Taraz [ABCHPT21] proved that this conjecture holds when all the trees have maximum degree $\leq c\frac{n}{\log n}$ for some constant $c>0$.
Janzer and Montgomery [JaMo24] have proved that there exists some $c>0$ such that the largest $cn$ trees can be packed into $K_n$.
Is it true that $f_k(n)\to \infty$ as $n\to \infty$? In particular, is it true that $f_4(n) \gg \log n$?
This conjecture was disproved by Rödl and Tuza [RoTu85], who proved that in fact $f_k(n)=\binom{k-1}{2}$ (for all sufficiently large $n$).
This is true. Pósa [Po76] proved that almost surely a random graph with $\geq Cn\log n$ edges is Hamiltonian for some large constant $C$, and Komlós and Szemerédi [KoSz83] proved that \[\geq \frac{1}{2}n\log n+\frac{1}{2}n\log\log n+w(n)n\] edges suffices, for any function $w$ which $\to \infty$ as $n\to \infty$.
This is now essentially completely understood: Johansson, Kahn, and Vu [JKV08] proved that the threshold is $\ell(n)\asymp n\log n$. The precise asymptotic was given by Kahn [Ka23], proving that the threshold is $\sim n\log n$ (also for the general problem over $r$-uniform hypergraphs).
This is true, and in fact $f(n) \ll 2^{n/2}$, which was proved independently by Green [Gr04] and Sapozhenko [Sa03]. In fact, both papers prove the stronger asymptotic $f(n) \sim c_n 2^{n/2}$, where $c_n$ takes on one of two values depending on the parity of $n$.
See [877] for the maximal case.
The answer is yes, proved by Sudakov and Verstraëte [SuVe08], who in fact proved that under the assumption of average degree $k$ and girth $>2s$ there are at least $\gg k^s$ many consecutive even integers which are cycle lengths in $G$.
Does there exist some constant $c>0$ such that \[\chi_L(G)+\chi_L(G^c)> n^{1/2+c}\] for every graph $G$ on $n$ vertices (where $G^c$ is the complement of $G$)?
The answer is no: Alon [Al92] proved that, for every $n$, there exists a graph $G$ on $n$ vertices such that \[\chi_L(G)+\chi_L(G^c)\ll (n\log n)^{1/2},\] where the implied constant is absolute.
Is it true that $f(n)\leq \frac{n}{2}+O(1)$?
See also [753].
Erdős believed this conjectured upper bound should hold even if we count equilateral triangles of any size.
The answer is yes: Bhowmick [Bh24] constructs a set of $n$ points in $\mathbb{R}^2$ such that $\lfloor\frac{n}{4}\rfloor$ distances occur at least $n+1$ times. More generally, they construct, for any $m$ and large $n$, a set of $n$ points such that $\lfloor \frac{n}{2(m+1)}\rfloor$ distances occur at least $n+m$ times.
Erdős and Sós proved that $c\geq 1/2$. Gyárfás and Lehel [GyLe95] proved \[\frac{1}{2}<c<\frac{3}{5}.\] (The example proving the upper bound is the set of the first $n$ Fibonacci numbers.)
Determine $z(n)$ for small values of $z(n)$. In particular is it true that $z(12)=4$?
In fact there do exist such graphs - Bhavik Mehta found computationally that there is exactly one (up to taking the complement) graph on $12$ vertices such that both $G$ and its complement are $K_4$-free with chromatic number $\geq 5$. This graph was explicitly checked to have cochromatic number $4$, and hence this proves that indeed $z(12)=4$.
The values of $z(n)$ are now known for $1\leq n\leq 19$: \[1,1,2,2,3,3,3,3,4,4,4,4,5,5,5,6,6,6,6.\] (The only significant difficulty here is proving $z(12)=4$ - the others follow from easy inductive arguments and the facts that $R(3)=6$ and $R(4)=18$.) It is unknown whether $z(20)=6$ or $7$.
Gimbel [Gi86] has shown that $z(n) \asymp \frac{n}{\log n}$.
Let $z(S_n)$ be the maximum value of $\zeta(G)$ over all graphs $G$ which can be embedded on $S_n$, the orientable surface of genus $n$. Determine the growth rate of $z(S_n)$.
If $G$ is a graph with chromatic number $\chi(G)=m$ then must $G$ contain a subgraph $H$ with \[\zeta(H) \gg \frac{m}{\log m}?\]
The answer is yes, proved by Alon, Krivelevich, and Sudakov [AKS97].
Must a graph with large chromatic number have large dichromatic number? Must a graph with large cochromatic number contain a graph with large dichromatic number?
Is it true that if $G$ has no $K_5$ and $\zeta(G)\geq 4$ then $\chi(G) \leq \zeta(G)+2$?
This has been disproved by Steiner [St24b], who constructed a graph $G$ with $\omega(G)=4$, $\zeta(G)=4$, and $\chi(G)=7$.
The answer is no, proved in a strong form by Vaughan [Va72], who showed that in fact \[\sum_{n\leq N} 1_A\ast 1_A\ast 1_A(n) = cN+o\left(\frac{N^{1/4}}{(\log N)^{1/2}}\right)\] is impossible. Vaughan proves a more general result that applies to any $h$-fold convolution, with different main terms permitted.
See also [572].
Give good estimates for $f(n;k,l)$ in the range $k<l\leq k^2/4$. For fixed $k$ and large $n$ is $f(n;k,l)$ a strictly monotone function of $l$?
The conjectured equality was proved for $n\geq 3k+3$ by Jiang [Ji04].
Curiously, in [Er69b] Erdős mentions this problem, but states that his conjectured equality for $g_k(n)$ was disproved (for general $k$) by Lewin, citing oral communication. Perhaps Lewin only disproved this for small $n$, or perhaps Lewin's disproof was simply incorrect.
Does, for every prime $p$, the density $\delta_p$ of integers with $h(n)=p$ exist? Does $\liminf h(n)=\infty$? Is it true that if $p$ is the greatest prime such that $p-1\mid n$ and $p>n^\epsilon$ then $h(n)=p$?
It is probably true that $h(n)=3$ for infinitely many $n$.
Is it true that \[f(n) = \left(\frac{1}{2}+o(1)\right)\frac{n}{\log n}?\]
The complementary bound \[f(n) \leq \left(\frac{1}{2}+o(1)\right)\frac{n}{\log n}\] was proved by Alon and Freiman [AlFr88], who chose $m$ as the least common multiple of $\{1,\ldots,s\}$ where $s$ is maximal such that $m\leq \frac{n^2}{20(\log n)^2}$.
Is it true that $H_k(n)/n^{1/2}\to \infty$? Or even $H_k(n) > n^{1/2+c}$ for some constant $c>0$?
The answer is yes, and in fact \[H_k(n) \gg_k n^{2/3},\] proved by Alon and Erdős [AlEr85]. We sketch their proof as follows: take a random subset $A'\subset A$, including each $n\in A'$ with probability $\asymp n^{-1/3}$. The number of non-trivial additive quadruples in $A$ is $\ll n^2$ and hence only $\ll n^{2/3}$ non-trivial additive quadruples remain in $A'$. Since the size of the random subset is $\gg n^{2/3}$, all of the remaining non-trivial additive quadruples can be removed by removing at most $\lvert A'\rvert/2$ (choosing the constants suitably).
Let $A\subset \mathbb{N}$ be an infinite set. We call $A$ proportionately dissociated if every finite $B\subset A$ contains a dissociated set of size $\gg \lvert B\rvert$.
Is every proportionately dissociated set the union of a finite number of dissociated sets?
Alon and Erdős write that it 'seems unlikely that [this] is also sufficient'. They also point out the same question can be asked replacing dissociated with Sidon (in the additive combinatorial sense).
How large must $n$ be (as a function of $r$) to ensure that there is such a family which achieves $n-3$ distinct sizes of sets?
Is it true that, if $\epsilon>0$ and $n$ is sufficiently large, whenever $m\leq (2-\epsilon)2^{n/2}$ the graph $G_\mathcal{F}$ has $<2^{n}$ many edges?
Is it true that if $G_{\mathcal{F}}$ has $\geq cm^2$ edges then $m\ll_c 2^{n/2}$?
Is it true that, for any $\epsilon>0$, there exists some $\delta>0$ such that if there are $>m^{2-\delta}$ edges then $m<(2+\epsilon)^{n/2}$?
For the first question we need to take $\epsilon>0$ since since if $n$ is even and $m=2^{n/2+1}$ one could take $\mathcal{F}$ to be all subsets of $\{1,\ldots,n/2\}$ together with $\{1,\ldots,n/2\}$ union all subsets of $\{n/2+1,\ldots,n\}$, which produces $2^{n}$ edges.
The third question was answered in the affirmative by Alon and Frankl [AlFr85], who proved that, for every $k\geq 1$, if $m=2^{(\frac{1}{k+1}+\delta)n}$ for some $\delta>0$ then the number of edges is \[< \left(1-\frac{1}{k}\right)\binom{m}{2}+O(m^{2-\Omega_k(\delta^{k+1})}).\] They also answer the second question in the negative, noting that if $\mathcal{F}$ is the family of sets which either intersect $\{n/2+1,\ldots,n\}$ in at most $1$ element or intersect $\{1,\ldots,n/2\}$ in at least $n/2-1$ elements then $m \gg n2^{n/2}$ and there are at least $2^{-5}\binom{m}{2}$ edges.
Finally, an affirmative answer to the first question follows from Theorem 1.4 and Corollary 1.5 of Alon, Das, Glebov, and Sudakov [ADGS15].
Does Bob have a winning strategy for $n\geq 3$? (Erdős believed the answer is yes.)
If we change the game so that Bob colours two edges after each edge that Alice colours, but now require Bob's largest clique to be strictly larger than Alice's, then does Bob have a winning strategy for $n>3$?
Finally, consider the game when Alice wins if the maximum degree of the red subgraph is larger than the maximum degree of the blue subgraph. Who wins?
Similarly, for the third game they prove that the set of $n$ for which Bob wins has density at least $2/3$, and prove the stronger statement that if Alice wins at $n$ then Bob wins at $n+1,n+2$.
When $k=2$ this was conjectured by Kneser and proved by Lovász [Lo78]. The general case was proved by Alon, Frankl, and Lovász [AFL86].
An affirmative answer to the first question implies an affirmative answer to the second.
Solymosi [So07] conjectured the answer to the second question is no. Cilleruelo and Granville [CiGr07] have observed that the answer to the second question is no conditional on the Bombieri-Lang conjecture.
What choice of such an $A$ minimises the number of integers $m\leq n$ not divisible by any $a\in A$? Is this minimised by letting $n\geq q_1>q_2>\cdots$ be the consecutive primes in decreasing order and choosing $A=\{q_1,\ldots,q_k\}$ where $k$ is maximal such that \[\sum_{i=1}^k\frac{1}{q_i}\leq C?\]
The answer is yes, proved by Sárközy and Szemerédi [SaSz94]. Ruzsa [Ru17] has constructed, for any function $w(x)\to \infty$, such a pair of sets with \[A(x)B(x)-x<w(x)\] for infinitely many $x$.
Similarly, can one always find a set $A\subset\{1,\ldots,N\}$ with this property of size $\geq (1-o(1))N$?
Selfridge constructed such a set with density $1/e-\epsilon$ for any $\epsilon>0$: let $p_1<\cdots<p_k$ be a sequence of large consecutive primes such that \[\sum_{i=1}^k\frac{1}{p_i}<1<\sum_{i=1}^{k+1}\frac{1}{p_i},\] and let $A$ be those integers divisible by exactly one of $p_1,\ldots,p_k$.
For the second question the set of integers with a prime factor $>N^{1/2}$ give an example of a set with size $\geq (\log 2)N$. Erdős could improve this constant slightly.
Estimate $g(n)$.
Estimate $f(n)$. In particular is it true that $f(n)\leq n^{1/2+o(1)}$?
Estimate $h(n)$.
Estimate $l(n)$. In particular, is it true that $l(n)n^{-1/2}\to \infty$? Is it true that $l(n)< n^{1-c}$ for some $c>0$?
See also [876].
Erdős [Er69] gave a simple proof that $F(n) \leq \pi(n)+n^{2/3}$: we define a graph with vertex set the union of those integers in $[1,n^{2/3}]$ with all primes $p\in (n^{2/3},n]$. We have an edge $u\sim v$ if and only if $uv\in A$. It is easy to see that every $m\leq n$ can be written as $uv$ where $u\leq n^{2/3}$ and $v$ is either prime or $\leq n^{2/3}$, and hence there are $\geq \lvert A\rvert$ many edges. This graph contains no path of length $3$ and hence must be a tree and have fewer edges than vertices, and we are done. This can be improved to give the upper bound mentioned by using a subset of integers in $[1,n^{2/3}]$.
More generally, one can ask for such an asymptotic for the size of sets such that no $a\in A$ divides the product of $r$ distinct other elements of $A$, with the exponent $2/3$ replaced by $\frac{2}{r+1}$.
See also [425].
This was solved by Raghavan [Ra25], who proved that \[g(n) \leq \pi(n)+\pi(n^{1/2})+O(n^{5/12+o(1)}),\] and also that \[g(n) \geq \pi(n)+\pi(n^{1/2})+\pi(n^{1/3})/3-O(1).\]
Estimate $g_k(n)$. In particular, is it true that \[g_k(n)=\frac{\log\log n}{\log n}n+(c+o(1))\frac{n}{(\log n)^2}\] for some constant $c$?
In particular the asymptotics of $g_k(n)$ are known; in this problem Erdős was asking about the second order terms. For $k=3$ he could prove the existence of some $0<c_1\leq c_2$ such that \[\frac{\log\log n}{\log n}n+c_1\frac{n}{(\log n)^2}\leq g_k(n)\leq \frac{\log\log n}{\log n}n+c_2\frac{n}{(\log n)^2}.\]
The special case $k=2$ is the subject of [425].
Estimate $f(d)$. In particular is it true that $f(d)=o(d^2)$?
Resolved by Alon, McDiarmid, and Reed [AMR91] who showed \[\frac{d^{4/3}}{(\log d)^{1/3}}\ll f(d) \ll d^{4/3}.\]
Estimate $t(n)$. In particular, is it true that $t(n)=o(n)$?
Resolved by Alon [Al91] who proved $t(n) \ll n^{2/3}\log n$.
Is it true that $\chi_L(G)=o(n)$ for almost all graphs on $n$ vertices?
The answer is yes: Alon [Al92] proved that in fact the random graph on $n$ vertices with edge probability $1/2$ has \[\chi_L(G) \ll \frac{\log\log n}{\log n}n\] almost surely. Alon, Krivelevich, and Sudakov [AKS99] improved this to \[\chi_L(G) \asymp \frac{n}{\log n}\] almost surely.
Is it true that for every $m\geq 1$, if $n$ is sufficiently large, any graph on $n$ vertices with $\geq n\log_2n$ edges contains a $O(1)$-balanced subgraph with $m$ vertices and $\gg m\log m$ edges (where the implied constants are absolute)?
Alon [Al08] proved this is actually false: for every $D>1$ and $n>10^5$ there is a graph $G$ with $\leq 2n$ vertices and $\geq 2n\log(2n)$ edges such that if $H$ is a $D$-balanced subgraph then $H$ has $\ll m(\sqrt{\log m}+\log D)$ many edges.
Estimate $f(n)$. In particular, is it true that $f((\log n)^2,n) \geq n^{1/2-o(1)}$? Is it true that $f((\log n)^3,n)\gg (\log n)^3$?
See also [805].
In particular, is there such a graph for $g(n)=(\log n)^3$?
Alon, Bucić, and Sudakov [ABS21] construct such a graph with \[g(n)\leq 2^{2^{(\log\log n)^{1/2+o(1)}}}.\] See also [804].
Resolved by Alon, Bukh, and Sudakov [ABS09], who proved that for any $A\subseteq \{1,\ldots,n\}$ with $\lvert A\rvert \leq n^{1/2}$ there exists some $B$ such that $A\subseteq B+B$ and \[\lvert B\rvert \ll \frac{\log\log n}{\log n}n^{1/2}.\]
See also [333].
Is it true that, if $G$ is a random graph on $n$ vertices with edge probability $1/2$, then \[\tau(G)=n-\alpha(G)\] almost surely?
This strong conjecture was disproved by Alon, Ruzsa, and Solymosi [ARS20], who constructed (for arbitrarily large $n$) a set of integers $A$ with $\lvert A\rvert=n$ and a graph $G$ with $\gg n^{5/3-o(1)}$ many edges such that \[\max(\lvert A+_GA\rvert,\lvert A\cdot_G A\rvert) \ll \lvert A\rvert^{4/3+o(1)}.\] Alon, Ruzsa, and Solymosi do prove, however, that if $A$ has size $n$ and $G$ has $m$ edges then \[\max(\lvert A+_GA\rvert,\lvert A\cdot_G A\rvert) \gg m^{3/2}n^{-7/4}.\]
Is it true that \[F_k(n)\sim n^2/8?\]
See also [810].
See also [809].
If $d_G(n)$ exists then determine the best possible value of $d_G(n)$.
The Kürschák competition in Hungary in 1986 asked students to prove that $d_{K_3}(n)$ exists. Kostochka proved that $d_{K_3}(n)=n/4$ is the best possible. Tuza proved that \[d_{C_4}(n) \leq \left(\frac{1}{4}-c\right)n\] for some constant $c>0$. Brightwell and Trotter proved that \[d_{C_6}(n) > (1-o(1))\frac{n}{6}.\]
Bucić and Sudakov [BuSu23] have proved \[h(n) \gg n^{5/12-o(1)}.\]
Such graphs are called degree $3$ critical. This conjecture was disproved by Narins, Pokrovskiy, and Szabó [NPS17], who proved that there are arbitrarily large such graphs with no cycle of length $23$.
It remains open whether this question has an affirmative answer if we restrict to even $k$.
Is it true that $H(n)=3$ infinitely often? (That is, $(2^n-1,3^n-1)=1$ infinitely often?)
Estimate $H(n)$. Is it true that there exists some constant $c>0$ such that, for all $\epsilon>0$, \[H(n) > \exp(n^{(c-\epsilon)/\log\log n})\] for infinitely many $n$ and \[H(n) < \exp(n^{(c+\epsilon)/\log\log n})\] for all large enough $n$?
Does a similar upper bound hold for the smallest $k$ such that $(k^n-1,2^n-1)=1$?
This conjecture would follow if we knew that, for every $\epsilon>0$, there are $\gg_\epsilon \frac{x}{\log x}$ many primes $p<x$ such that all prime factors of $p-1$ are $<p^\epsilon$.
See also [416].
Is it true that $h(x)>x^{2-o(1)}$?
A similar question can be asked if we replace the condition $(a,b)=1$ with the condition that $a$ and $b$ are squarefree.
Erdős suggested that as $C\to \infty$ only divisors at most $\epsilon n$ need to be used, where $\epsilon \to 0$.
See also [18].
Determine $n_k$.
This was disproved by Alon [Al85], who proved, for example, that there exists some absolute constant $C>0$ such that if $r\geq C$ and $k\geq Cr$ then there exists an $r$-uniform hypergraph with chromatic number $\geq k$ with at most \[\leq (7/8)^r\binom{(r-1)(k-1)+1}{r}\] many edges.
In general, Alon gave an upper bound for the minimal number of edges using Turán numbers. Using known bounds for Turán numbers then suffices to disprove this conjecture for all $r\geq 4$. The validity of this conjecture for $r=3$ remains open.
If $m(r,k)$ denotes the minimal number of edges of any $r$-uniform hypergraph with chromatic number $>k$ then Akolzin and Shabanov [AkSh16] have proved \[\frac{r}{\log r}k^r \ll m(r,k) \ll (r^3\log r) k^r,\] where the implied constants are absolute. Cherkashin and Petrov [ChPe20] have proved that, for fixed $r$, $m(r,k)/k^r$ converges to some limit as $k\to \infty$.
This was solved by Erdős and Lovász [ErLo75], who proved in particular that there is a vertex contained in at least \[\frac{2^{r-1}}{4r}\] many edges.
They do not specify what is meant by $3$-critical. One definition in the literature is: a hypergraph is $3$-critical if there is a set of $3$ vertices which intersects every edge, but no such set of size $2$, and yet for any edge $e$ there is a pair of vertices which intersects every edge except $e$. Raphael Steiner observes that a $3$-critical hypergraph in this sense has bounded size, so this problem would be a finite computation, and perhaps is not what they meant.
An alternative definition is that a hypergraph is $3$-critical if it has chromatic number $3$, but its chromatic number becomes $2$ after deleting any edge or vertex.
This is equivalent to asking whether there exists $k>2$ such that the chromatic number of the Johnson graph $J(2k,k)$ is $k+1$ (it is always at least $k+1$ and at most $2k$). The chromatic numbers listed at this website show that this is false for $3\leq k\leq 8$.
Erdős and Lovász [ErLo75] proved that there must be two edges which meet in $\gg \frac{r}{\log r}$ many vertices.
What is $A_3$?
The analogous question with $A-A$ in place of $A+A$ is simpler, and there the maximal size is $\sim N^{1/2}$, as proved by Cilleruelo.
Bui, Pratt, and Zaharescu [BPZ24] proved that the distribution of $t_n$ continues to follow $P(n)$, in that for any fixed $c\in (0,1]$ \[\lim_{x\to \infty}\frac{\lvert \{ n\leq x : t_n\leq n^c\}\rvert}{x} = \lim_{x\to \infty}\frac{\lvert \{ n\leq x : P(n)\leq n^c\}\rvert}{x}.\] They also prove that for at least $x^{1-o(1)}$ many $n\leq x$ we have \[t_n \leq \exp(O(\sqrt{\log n\log\log n}))\] and for all non-square $n$ \[t_n \gg (\log\log n)^{6/5}(\log\log\log n)^{-1/5}.\]
See also [437].
That is, is it true that, in any 2-colouring of the square numbers, every sufficiently large $n\in \mathbb{N}$ can be written as a monochromatic sum of distinct squares?
Indeed, one proves (by induction) the stronger fact that such a representation always exists, and moreover if $n$ is even then all the summands can be taken to be even: if $n=2m$ we are done applying the inductive hypothesis to $m$. Otherwise if $n$ is odd then let $3^k$ be the largest power of $3$ which is $\leq n$ and apply the inductive hypothesis to $n-3^k$ (which is even).
See also [123].
Is it true that $A$ is the union of a finite number of sets where no three are on a line?
Is it true that $A$ is the union of a finite number of sets which contain no three-term arithmetic progression?
Both Erdős and Singmaster believed the answer to this question is no, and in fact that there exists an absolute upper bound on the number of solutions.
Matomäki, Radziwill, Shao, Tao, and Teräväinen [MRSTT22] have proved that there are always at most two solutions if we restrict $k$ to \[k\geq \exp((\log n)^{2/3+\epsilon}),\] assuming $a$ is sufficiently large depending on $\epsilon>0$.
See also [677].
Is it true that $r(x)\to \infty$? Or even $r(x)/\log x \to \infty$?
This is probably false, since Hensley and Richards [HeRi73] have shown that this is false assuming the Hardy-Littlewood prime tuples conjecture.
Erdős [Er85c] reports Straus as remarking that the 'correct way' of stating this conjecture would have been \[\pi(x+y) \leq \pi(x)+2\pi(y/2).\] Clark and Jarvis [ClJa01] have shown this is also incompatible with the prime tuples conjecture.
In [Er85c] Erdős conjectures the weaker result (which in particular follows from the conjecture of Straus) that \[\pi(x+y) \leq \pi(x)+\pi(y)+O\left(\frac{y}{(\log y)^2}\right),\] which the Hensley and Richards result shows (conditionally) would be best possible. Richards conjectured that this is false.
Erdős and Richards further conjectured that the original inequality is true almost always - that is, the set of $x$ such that $\pi(x+y)\leq \pi(x)+\pi(y)$ for all $y<x$ has density $1$. They could only prove that this set has positive lower density.
They also conjectured that for every $x$ the inequality $\pi(x+y)\leq \pi(x)+\pi(y)$ is true provided $y \gg (\log x)^C$ for some large constant $C>0$.
Hardy and Littlewood proved \[\pi(x+y) \leq \pi(x)+O(\pi(y)).\] The best known in this direction is a result of Montgomery and Vaughan [MoVa73], which shows \[\pi(x+y) \leq \pi(x)+2\frac{y}{\log y}.\]
Estimate $f_k(N)$.
The analogous question with natural density in place of logarithmic density (that is, we measure $\lvert A\rvert$ in place of $\sum_{n\in A}\frac{1}{n}$) is the subject of [536]. In particular Erdős [Er70] has constructed $A\subseteq \{1,\ldots,N\}$ with $\lvert A\rvert \gg N$ where no four have the same pairwise least common multiple, and hence the interest of the natural density problem is the $k=3$ case.
A related combinatorial problem is asked at [857].
Estimate $m(n,k)$, or even better, give an asymptotic formula.
This is sometimes known as the weak sunflower problem (see [20] for the strong sunflower problem).
When $k=3$ this is strongly connected to the cap set problem (finding the maximal size of subsets of $\mathbb{F}_3^n$ with no three-term arithmetic progressions), as observed by Alon, Shpilka, and Umans [ASU13]). Naslund and Sawin [NaSa17] have proved that \[m(n,3) \leq (3/2^{2/3})^{(1+o(1))n}.\]
An example of such a set $A$ is the set of all integers in $[N^{1/2},N]$ divisible by some prime $>N^{1/2}$.
See also [143].
Do there exist constants $c_1,c_2>0$ such that \[d_t \sim \frac{c_1}{(\log t)^{c_2}}\] as $t\to \infty$?
Estimate $h(n)$.
While $A(N)$ has not been completely determined, both of these questions are now settled, the first positively and the second negatively. The current best bounds are (for large $N$) \[2^{1.16f(N)}\leq A(N) \leq 2^{6.442f(N)}.\] The lower bound is due to Saxton and Thomason [SaTh15], the upper bound is due to Kohayakawa, Lee, Rödl, and Samotij [KLRS].
See also [862].
Similarly, let $B\subseteq \{1,\ldots,N\}$ be a set of maximal size such that there are at most $r$ solutions to $n=a-b$ for any $n$.
If $\lvert A\rvert\sim c_rN^{1/2}$ as $N\to \infty$ and $\lvert B\rvert \sim c_r'N^{1/2}$ as $N\to \infty$ then is it true that $c_r\neq c_r'$ for $r\geq 2$? Is it true that $c_r'<c_r$?
It is true that $c_1=c_1'$, and the classical bound on the size of Sidon sets (see [30]) implies $c_1=c_1'=1$.
For the analogous question with $n=a-b$ they prove that $\lvert A\rvert\sim N^{1/2}$.
This is a weaker form of [840].
It is a classical folklore fact that if $A\subseteq \{1,\ldots,2N\}$ has size $\geq N+2$ then there are distinct $a,b\in A$ such that $a+b\in A$, which establishes the $k=2$ case.
In general, one can define $f_k(N)$ to be minimal such that if $A\subseteq \{1,\ldots,N\}$ has size at least $f_k(N)$ then there are $k$ distinct $a_i\in A$ such that all $\binom{k}{2}$ pairwise sums are elements of $A$. Erdős and Sós conjectured that \[f_k(N)\sim \frac{1}{2}\left(1+\sum_{1\leq r\leq k-2}\frac{1}{4^r}\right) N,\] and a similar example shows that this would be best possible.
Choi, Erdős, and Szemerédi [CES75] have proved that, for all $k\geq 3$, there exists $\epsilon_k>0$ such that (for large enough $N$) \[f_k(N)\leq \left(\frac{2}{3}-\epsilon_k\right)N.\]
Estimate $g_k(N)$.
As an example, taking $A$ to be the set of all odd integers and the powers of $2$ shows that $g_5(N)\gg \log N$ for some $c>0$.
What if $1_A\ast 1_A(n) >\epsilon \log n$ (for all large $n$, for arbitrary fixed $\epsilon>0$)?
The game ends when no legal move is possible. One player wants the game to last as long as possible, the other wants the game to end quickly. How long can the game be guaranteed to last for?
At least $\epsilon n$ moves? (For $\epsilon>0$ and $n$ sufficiently large.) At least $(1-\epsilon)\frac{n}{2}$ moves?
This type of game is known as a saturation game.
See also [186] and [789]. For an infinite version of this problem see [875].
Erdős [Er62c] proved that a sum-free set has density zero. Deshouillers, Erdős, and Melfi [DEM99] constructed a sum-free set that grows like $a_n\sim n^{3+o(1)}$.
Luczak and Schoen [LuSc00] have proved that, for all large $N$, \[\lvert A\cap [1,N]\rvert\ll (N\log N)^{1/2},\] and that there exists a sum-free set $B$ such that \[\lvert B\cap [1,N]\rvert \gg \frac{N^{1/2}}{(\log N)^{1/2+o(1)}}\] for all large $N$.
See also [790].
See [748] for the non-maximal case.
Is it true that, for almost all $n$, \[f(n)=o(n\log\log n)\] and \[F(n) \gg n\log\log n?\] Is it true that \[\max_{n\leq x}f(n)\sim \frac{x\log x}{\log\log x}?\] Is it true that (for all $x$, or perhaps just for all large $x$) \[\max_{n\leq x}f(n)=\max_{n\leq x}F(n)?\] Find an asymptotic formula for the number of $n<x$ such that $f(n)=F(n)$. Find an asymptotic formula for \[H(x)=\sum_{n<x}\frac{f(n)}{n}.\] Is it true that \[H(x) \ll x\log\log\log\log x?\]
It is trivial that $f(n)\leq F(n)$ for all $n$. It may be true that, for almost all $n$, \[F(n)\sim \frac{1}{2}n\log\log n.\]
Erdős notes that $f(n)/n$ 'almost behaves as a conventional additive function', but unusually $f(n)/n$ does not have a mean value - indeed, \[\limsup \frac{1}{x}\sum_{n<x}\frac{f(n)}{n}=\infty\] but \[\liminf \frac{1}{x}\sum_{n<x}\frac{f(n)}{n}<\infty.\] Erdős [Er84e] proved that \[x\log\log\log\log x\ll H(x) \ll x\log\log\log x.\]
Hegyvári, Hennecart, and Plagne [HHP07] showed the answer is yes for $k=2$ (in fact with $b_{n+1}-b_n\leq 2$ for large $n$) but no for $k\geq 3$.
The proof that $b_{n+1}-b_n\leq 2$ for $k=2$ is trivial, since clearly all odd numbers in $A+A$ must be the sum of two distinct elements from $A$.
In [Er98] Erdős reports (but gives no reference) that Sándor has proved that $\lvert A\rvert=(1-o(1))\log_2 n$ is achievable, taking $A=\{ 2^i+m2^m : 0\leq i<m\}$ and $n=2^{m-1}+m2^m$. I have chosen to leave this problem as 'open' until a reference for this claim (or an alternative proof) can be found.
See also [13].
Is it true that if $\lvert A\rvert >\frac{2}{3}n$ then $G(A)$ contains all odd cycles of length $\leq \frac{n}{3}+1$?
Is it true that, for every $\ell\geq 1$, if $n$ is sufficiently large and $\lvert A\rvert>\frac{2}{3}n$ then $G(A)$ must contain a complete $(1,\ell,\ell)$ triparite graph on $2\ell+1$ vertices?
It is a classical fact that \[\limsup_{n\to \infty}\omega(n)\frac{\log\log n}{\log n}=1.\]
This is unknown even for $k=2$ - that is, is it true that in every interval of $6$ (sufficiently large) consecutive integers there must exist one with at least $3$ prime factors?
In particular, is this always possible if there are no non-trivial solutions to $(b_i,b_j)=b_k$?
One can ask a similar question for sequences of real numbers, as in [143].
Is it true that there must exist a finite colouring of $\mathbb{N}$ with no monochromatic solutions to $a-b\in A$?
Katznelson observed that a positive solution to the problem follows from the answer to [464], which yields an irrational $\theta$ and $\delta>0$ such that $\inf_k \| \theta n_k\|>\delta$.
Indeed, given such a $\theta$ a colouring of $\mathbb{N}$ using $\ll \delta^{-1}$ colours lacking any solution to $a-b\in A$ can be produced by dividing $\mathbb{R}/\mathbb{Z}$ into disjoint intervals of length $\leq \delta$ and then colouring $n$ according to which interval $\| \theta n\|$ belongs to.
In particular, the solution to [464] implies the answer to this question is yes, with the best known quantitative bound, due to Peres and Schlag [PeSc10], being that there is a colouring with no solutions using at most \[\ll \epsilon^{-1}\log(1/\epsilon)\] colours.
It is known that $m(2)=3$, $m(3)=7$, and $m(4)=23$. Erdős proved \[2^n \ll m(n) \ll n^2 2^n\] (the lower bound in [Er63b] and the upper bound in [Er64e]). Erdős conjectured that $m(n)/2^n\to \infty$, which was proved by Beck [Be78], who proved \[n^{1/3}2^n \ll m(n).\] Radhakrishnan and Srinivasan [RaSr00] improved this to \[\sqrt{\frac{n}{\log n}}2^n \ll m(n).\]
Is it true that if $t>n$ then $t\geq n+p$?
In [Er82e] Erdős writes that he and Sós proved some special cases of this and the full conjecture was proved by Wilson, but I cannot find either reference.
In general, one can ask what the possible values of $t$ are, for a given $n$.
If $n\geq 2$ does every connected set in $\mathbb{R}^n$ contain more than $2^{\aleph_0}$ many connected subsets?
Is there a function $f$ such that $f(x)/x\to \infty$ as $x\to \infty$ such that, for all large $C$, if $G$ is a graph with $n$ vertices and $e\geq Cn$ edges then \[\hat{R}(G) > f(C) e?\]
Prove that there exists some $c>0$ such that \[h(n) \sim c \left(\frac{n}{\log n}\right)^{1/2}\] as $n\to \infty$.
Bollobás proved this when $m=4$ - in fact he showed that every graph with $n$ vertices and $2n-1$ edges contains two points joined by $4$ edge-disjoint paths.
The answer is yes, as proved by Thomassen [Th74].
Is it true that \[f_k(n) \gg_k n^2?\] Is it true that \[f_6(n)\sim n^2/4?\] More generally, is it true that, for $k\geq 6$, \[f_k(n) \sim \frac{1}{2}\left(1-\frac{1}{\lfloor k/3\rfloor}\right)n^2?\]
Erdős [Er69b] observed that Dirac's construction generalises to show that, if $3\mid k$, there are infinitely many values of $n$ (those of the shape $mk/3$ where $m$ is odd) such that \[f_k(n) \geq \frac{1}{2}\left(1-\frac{1}{k/3}\right)n^2 + n.\]
Toft [To70] proved that $f_k(n)\gg_k n^2$ for $k\geq 4$.
Constructions of Stiebitz [St87] show that, for $k\geq 6$, there exist infinitely many values of $n$ such that \[f_k(n) \geq \frac{1}{2}\left(1-\frac{1}{\lfloor k/3\rfloor+\delta_k}\right)n^2\] where $\delta_k=0$ if $k\equiv 0\pmod{3}$, $\delta_k=1/7$ if $k\equiv 1\pmod{3}$, and $\delta_k\equiv 24/69$ if $k\equiv 2\pmod{3}$, which disproves Erdős' conjectured asympotic for $k\not\equiv 0\pmod{3}$.
Stiebitz also proved the general upper bound \[f_k(n) < \mathrm{ex}(n;K_{k-1})\sim \frac{1}{2}\left(1-\frac{1}{k-2}\right)n^2\] for large $n$. Luo, Ma, and Yang [LMY23] have improved this upper bound to \[f_k(n) \leq \frac{1}{2}\left(1-\frac{1}{k-2}-\frac{1}{36(k-1)^2}+o(1)\right)n^2\]
Is there a graph with $\aleph_{\omega+1}$ vertices and chromatic number $\aleph_1$ such that every subgraph on $\aleph_\omega$ vertices has chromatic number $\aleph_0$?
What if instead we ask for $G$ to have chromatic number $\aleph_1$?
Erdős and Hajnal showed this does not generalise to higher cardinals - they (see [Er69b]) constructed a set on $\omega_1^2$ with chromatic number $\aleph_1$ such that every strictly smaller subgraph has chromatic number $\leq \aleph_0$ as follows: the vertices of $G$ are the pairs $(x_\alpha,y_\beta)$ for $1\leq \alpha,\beta <\omega_1$, ordered lexicographically. We connect $(x_{\alpha_1},y_{\beta_1})$ and $(x_{\alpha_2},y_{\beta_2})$ if and only if $\alpha_1<\alpha_2$ and $\beta_1<\beta_2$.
A similar construction produces a graph on $\omega_2^2$ with chromatic number $\aleph_2$ such that every smaller subgraph has chromatic number $\leq \aleph_1$.
Is it true that, for $k\geq 4$, \[g_k(n) \gg \frac{n^{1-\frac{1}{k-1}}}{(\log n)^c}\] for some constant $c>0$?
The lower bound $R(4,m) \gg m^3/(\log m)^4$ of Mattheus and Verstraete [MaVe23] (see [166]) implies \[g_4(n) \gg \frac{n^{2/3}}{(\log n)^{4/3}}.\] In general it is known (see [BoKe10]) that \[R(k,m)\gg (\log m)^{-O_k(1)}m^{\frac{k+1}{2}}\] which implies \[g_k(n) \gg \frac{n^{1-\frac{2}{k+1}}}{(\log n)^{c_k}}.\]
This is true, and was proved by Folkman [Fo70b].
See also [73].
In other words, this question asks whether $R(3,3,m) \ll m^{3-c}$ for some $c>0$. This was disproved by Alon and Rödl [AlRo05], who proved that \[\frac{1}{(\log m)^{4+o(1)}}m^3 \ll R(3,3,m) \ll \frac{\log\log m}{(\log m)^2}m^3.\] As reported in [AlRo05] Sudakov has observed that the $\log\log m$ in the upper bound can be removed.
See also [553].
Since each $H_k$ is 2-degenerate this is a special case of [146].
This was disproved by Spencer [Sp71], who proved that in fact \[g(n) > n-\log_2 n-O(1).\]
See also [775].
Erdős also asked whether infinitely many such $n$ even exist, but Meza has observed that this follows immediately from Schinzel's result [Sc67b] that for infinitely many $n$ the largest prime factor of $n(n+1)$ is at most $n^{O(1/\log\log n)}$.
Estimate $S(k)$ - in particular, is it true that $S(k)\geq k^{1-o(1)}$?
It is trivial that $S(k)\leq k$ since, for example, one can take $n\equiv 1\pmod{k!}$. The best bound on large gaps between primes due to Ford, Green, Konyagin, Maynard, and Tao [FGKMT18] (see [4]) implies \[S(k) \ll k \frac{\log\log\log k}{\log\log k\log\log\log\log k}.\]
Estimate $h_t(d)$.
It is easy to see that $h_t(d)\leq 2d^t$ always and $h_1(d)=d+1$.
Erdős and Nešetřil and Bermond, Bond, Paoli, and Peyrat [BBPP83] independently conjectured that $h_2(d) \leq \tfrac{5}{4}d^2+1$, with equality for even $d$. This was proved by Chung, Gyárfás, Tuza, and Trotter [CGTT90].
Cambie, Cames van Batenburg, de Joannis de Verclos, and Kang [CCJK22] conjectured that \[h_3(d) \leq d^3-d^2+d+2,\] with equality if and only if $d=p^k+1$ for some prime power $p^k$, and proved that $h_3(3)=23$. They also conjecture that, for all $t\geq 3$, $h_t(d)\geq (1-o(1))d^t$ for infinitely many $d$ and $h_t(d)\leq (1+o(1))d^t$ for all $d$ (where the $o(1)$ term $\to 0$ as $d\to \infty$).
The same authors prove that, if $t$ is large, then there are infinitely many $d$ such that $h_t(d) \geq 0.629^td^t$, and that for all $t\geq 1$ we have \[h_t(d) \leq \tfrac{3}{2}d^t+1.\]
If $\ell\geq 2$ then is \[\lim_{n\to \infty}\frac{Q_2(n(n+1)\cdots(n+\ell))}{n^{\ell+1}}=0?\]
A result of Mahler implies, for every $\ell\geq 1$, \[\limsup_{n\to \infty}\frac{Q_2(n(n+1)\cdots(n+\ell))}{n^2}\geq 1.\] All these questions can be asked replacing $Q_2$ by $Q_r$ for $r>2$, only keeping those prime powers with exponent $\geq r$.
Without the coprimality condition this is easy, since if $a,a+d,\ldots,a+(k-1)d$ is an arithmetic progression of powerful numbers then so too is \[a(a+kd)^2,\ldots,(a+(k-1)d)(a+kd)^2,(a+kd)^3.\] Beginning with $k=2$ and an arbitrary pair of powerful numbers one can construct arbitrarily long arithmetic progressions of powerful numbers.
One can similarly ask for coprime arithmetic progressions in the $r$-powerful numbers (i.e. if $p\mid n$ then $p^r\mid n$). Erdős [Er76d] conjectured that when $r\geq 4$ there do not exist infinitely many such progressions of length $3$, and when $r=3$ there are infinitely many progressions of length $3$ but only finitely many of length $4$.
Bajpai, Bennett, and Chan [BBC24] proved that there are infinitely many four-term progressions of coprime powerful numbers, and infinitely many three-term progressions of coprime $3$-powerful numbers. They also show that there exist only finite many three-term coprime progressions when $r\geq 4$ assuming the abc conjecture.
Are there only finitely many three-term progressions of consecutive terms $n_k,n_{k+1},n_{k+2}$?
If $r\geq 4$ then can the sum of $r-2$ coprime $r$-powerful numbers ever be itself $r$-powerful? Are there at most finitely many such solutions?
Are there infinitely many triples of coprime $3$-powerful numbers $a,b,c$ such that $a+b=c$?
Euler had conjectured that the sum of $k-1$ many $k$th powers is never a $k$th power, but this is false for $k=5$, as Lander and Parkin [LaPa67] found \[27^5+84^5+110^5+133^5=144^5.\]
Cambie has found several examples of the sum of $r-2$ coprime $r$-powerful numbers being itself $r$-powerful. For example when $r=5$ \[3^761^5=2^83^{10}5^7+2^{12}23^6+11^513^5.\] Cambie has also found solutions when $r=7$ or $r=8$ (the latter even with the sum of $5$ $8$-powerful numbers being $8$-powerful).
It does not seem to even be known if all large integers are the sum of at most $r$ many $r$-powerful numbers (in [Er76d] Erdős claims this follows from a simple counting argument, but Schinzel pointed out he made a mistake).
Heath-Brown [He88] has proved that all large numbers are the sum of at most three $2$-powerful numbers.
Let $k\geq 4$ and $r\geq 1$. Must there exist a graph $G$ with chromatic number $k$ such that every vertex is critical, yet every critical set of edges has size $>r$?
This was conjectured by Dirac in 1970 for $k\geq 4$ and $r=1$. Dirac's conjecture was proved, for $k=5$, by Brown [Br92]. Lattanzio [La02] proved there exist such graphs for all $k$ such that $k-1$ is not prime. Independently, Jensen [Je02] gave an alternative construction for all $k\geq 5$. The case $k=4$ and $r=1$ remains open.
Martinsson and Steiner [MaSt25] proved this is true for every $r\geq 1$ if $k$ is sufficiently large, depending on $r$.
This is Problem 91 in the graph problems collection.